You are on page 1of 152
downloaded from jeemain. guru CHAPTER | = ayy a Trigonometric Ratios and Identities . VVVVVVVVVVVV Measurement of Angles Trigonometric Functions Problems Based on Trigonometric Identities Trigonometric Ratios for Complementary and Supplementary Angles Trigonometric Ratios for Compound Angles Transformation Formulae Trigonometric Ratios of Multiples and Sub-Multiple Angles Values of Trigonometric Ratios of Standard Angles Sum of Sines or Cosines of n Angles in A.P, Conditional Identities Some Important Results and their Applications Important Inequalities downloaded from jeemain.guru 22 ‘Trigonometry MEASUREMENT OF ANGLES Angles in Trigonometry In trigonometry, the idea of angle is more general; it may be positive or negative and has any magnitude (Fig.2.1). ° A Fig. 2.1 In trigonometry, as in case of geometry, the measure of angle is the amount of rotation from the direction of ray of the angle to the other. The initial and final positions of the revolving ray are respectively called the ial side (arm) and terminal side (arm), and thé revolving line is called the generating line or the radius vector. For example, if OA and OB are the initial and final positions of the revolving ray, then the angle formed will be ZAOB. Angles Exceeding 360° In geometry, we confine ourselves to angles from 0° to 360°. But there may be problems in which rotation involves more than one revolution, for example; the rotation of a flywheel. In trigonometry, we generalise the concept of angle to angles greater than 360°. This angle can be formed in the following way: The revolving line (radius vector) starts from the initial position OA and makes n complete revolutions in anticlockwise direction and also a further angle er in the same direction. We then have a certain angle B, given by B, = 360° x7+ ar, where 0° < or< 360° and n isa positive integer or zero. Thus, there are infinitely many 8, angles with initial side OA and final side OB, For example, fly = 0 8, = 360° + a By = 720° + a, ete. a=fy 360° + a= fy ° A im Fig. 2.2 Sign of Angles Angles formed by anticlockwise rotation of the radius vector are taken as positive, whereas angles formed by clockwise rotation of the radius vector are taken as negative. downloaded from jeemain. guru ‘Trigonometric Ratios and Identities 2.3 +1360" - 0) ° A A Fig. 2.3 Relation between Degree and Radian Radian is a constant angle. One radian is the angle subtended by an arc of a circle at the centre, It is equal to arc/radius. It is expressed as rad. Fig.2.4 Consider a circle with centre O and radius r. Let A bea point on the citcle, Join OA and cut off an arc of length equal to the radius of the circle, Then, ZAOP= 1 rad, Produce AO to meet the circle at B. => ZAOB=astraight angle =2 right angles We know that the angles at the centre of a circle are proportional to the arcs subtending them. =p AQP __arcAP ZAOB arcAPB ZAOP c 1. = Fright angles ~ xr [ are PB =1 iremerence => Zrop= 2tghangles jr. 180° ™ m Hence, | rad= ie = mrad = 180°, 1 Note: ‘* When an angle is expressed in radian, the word radian is generally omitted, © 1°=60" (60 min) and 1’ =60" (60 sec) * Since 180°= xrad. Therefore, 1° = 1/180 rad. en x30 =% Hence, 30°= 5x30 == rad, 49°= x45 == rad, 180 4 downloaded from jeemain. guru 24 Trigonometry 90°= 2x90 == rad, etc. 180 2 © We have xrad=180° = Irad= Ee (2x7) =57°16/22" (approx) a \2 - iin pete wan © 180°= trad= 1 ha [ey * Sum of imerior angles on convex polygon of n sides is (n—2)n rad. Express 45° 20/10” in rad measure (r =3.1415), )pad=0 01746 rad. nO pain = 10 — degrees = = degrees Sol. 10’ = 60 60x60 are 360 I 20 20’= = degrees = +d legrees = — degrees 3 ase (+s eda om 16200+1 +120 _ 16321 360 360 360 Now 1632! - 6321 1632! 416 51274.054 =0.79 rad 360 360 360 180, 64800 (BREE. express 1.2 rad in degree measure. * Sol. (.29%=1.2% HO degrees = 1.2 x OX? [2-2 (arex)] = = 68.7272 = 68° (.7272.x 60)'=68° (43.63) =68° 43’ (.63 x 60)” = 68° (43’37.8") PRETEEN Find thetength ofan arcofacircle of radius 5 em subtending a central angle measuring 15°. Sol. Let s be the length of the arc subtending an angle 6* at the centre of a circle of radius r. Then, @=s/r. * oi Here,r=5em and 6=15°= [15x] =/ = 180, 12, snes Sa S225 o5- em r 125 12 Find in degrees the angle subtended at the centre of a circle of diameter 50 cm by an arc of length 11 em. Sol. Here,r=25 cm ands= 1] cm. : . (f(a (et ¥ 25 23 x = (M4 25° 22 = (3) bss) fe) 25°12" downloaded from jeemain. guru Trigonomettic Ratios and Identities 2.5 Ifares of same length in two circles subtend angles of 60° and 75° at their centres, find the ratios of their radii. Sol, Let rand ry be the radii of the given circles and let their ares of same length ssubtend angles of 60° np and 75? at their centres. t ra\h * Ce G —| =/— Sx No -aat~ (sox ) ( ) 73° (155) =/= ny 1g0J (3) am 180) ~ (32 Bre Lag SS ig Eye any wes Bye y dy oS ermsed 3, 12 mh 3 12 3772 Hence, rj :7=5:4. . a Assuming the distance of earth from the moon to be 38,400 km and the angle subtended by the moon at the eye of a person on earth to be 31’, find the diameter of the moon. Sol, A c ° @ Fig. 2.5 Let AB be the diameter of the moon and O be the observer. 31 on ex rad ‘ Given ZAOB= 31 = = x ra Since the angle subtended by the moon is very small, its diameter will be approximately equal to the small arc of a circle whose centre is the eye of the observer and the radius is the distance of the earth from the moon. Also the moon subtends an angle of 31’ at the centre of this circle. r 31 e _ AB = O=—, therefore = x = = r 60” 180 ~ 38400 3122 8 =— x x 38, 400 = 3464 — km = AB 75180 @ Find the angle between the minute hand and the hour hand of a clock when the time is7:20AM. Sol. We know that the hour hand completes one rotation in 12 hr, while the minute hand completés one rotation in 60 min. Therefore, the angle traced by the hour hand in 12 hr = 360° Angle traced by the hour hand in 7 hr 20 min, i.e., 2 hr= (2x 2) = 220° ‘Aso, the angle traced by the minute hand in 60 min = 360° = 1202 Hence, the required angle between the two hands = 220° — 120° = 100°. The angle traced by the minute hand in 20 mi downloaded from jeemain.guru 26 Trigonometry For each natural number, 4, let C, denote a circle with radius k centimeters and centre at origin O. On the circle C, a particle moves & centimetres in the counter-clockwise direction, After completing its motion on C,, the particle moves to C,,; in the radial direction. The motion of the particle continues in this manner. The particle starts at (1, 0). If the particle crosses the positive direction of x-axis for the first time on the circle C,, then find the value of, Sol. Fig. 2.6 The motion of the particle on the first four circles is shown with bold line in Fig. 2.6, Note that on every circle the particle travels just | rad, The particle crosses the positive direction of x-axis first time on C,, wheré nis the least positive integer such that n > 2 => n= 7. 1, A horse is tied to a post by a rope. If the horse moves along a circular path always keeping the rope tight and describes 88 m when it has traced out 72° at the centre, find the length of the rope. 2. Ifthe angular diameter of the moon is 30’, how far from the eye a coin of diameter 2.2.m can be kept to hide the moon? 3. Find in degrees and radians the angle between the hour hand and the minute hand ofa clock at half past three, 4. There is an equilateral triangle with side 4 and a circle with the centre on the one of the vertex of that triangle. The arc of that circle divides the triangle into two parts of equal area. How long is the radius of the circle? TRIGONOMETRIC FUNCTIONS Trigonometric Functions of Acute Angles ‘An angle whose measure is greater than 0° but less than 90° is called an acute angle. Consider a right-angled triangle ABC with right angle at B. The side opposite to the right angle is called the hypotenuse, side opposite to angle A is called the perpendicular for angle A and side opposite to the third angle is called the base for angle A. downloaded from jeemain. guru Trigonometric Ratios and identities 2.7 ew ¢ h Pp » p rr A Fig. 2.7 ‘The ratio of any two sides of the triangle depends only on measure of angle A, for if we take a larger and smaller right angte triangles as shown in Fig. 2.7, we have : : £ =~ (as these triangles are similar). bp Thus, the ratio of the lengths of any two sides of a triangle is completely determined by angle 4 alone and is independent of the size of the triangle. There are six possible ratios that can be formed from the three sides of a right-angled triangle. Each of them has been given a name as follows. Definitions ) sina= 2 p ni) i 22 @ sina= 2 Gi) cosa= > i namo (iv) cota= 2 (v) seca= z (vi) cosec A= o The abbreviations stand for sine, cosine, tangent, cotangent, secant, and cosecant of A, respectively. These functions of angle A are called trigonometrical functions or trigonomettical ratios. The circumference of a circle circumscribing an equilateral triangle is 247 units. Find the area of the circle inscribed in the equilateral triangle. Sol. 24R=247(Ris the radius of circumeircle) R=12 sin 30° = * (ris the radius of incircle) ‘Therefore, area of incircle = m? = 367 Fig.28 In triangle ABC, BC=8, CA =6 and AB =10. Aline regions of equal area is perpendicular to.AB at the point X. Then find the value of BX/¥2. iding the triangle ABC into two ¢ Sol. Fromthe figure, 2( 22) = 8X6 34 z) @ a . xxxtan B=24 vx 3=24 4 a ‘ Y=32>x=4V2 i. x downloaded from: jeemain. guru 28 ‘Trigonometry Let PQ and RS be tangents at the extremities of the diameter PR ofa circle of radius. IPS and RQ intersect at a point X on the circumference of the circle, then prove that 27 POXRS s Sol, From the figure, we have re =tan(-@=cot8. a and “5 = tan 6 ‘ % PR S aE aN = xt =I a PR PR pO Sp => (PRY = POX PS = (2rP=POXPS = 2r= POXPS Fig. 2.10 ‘Two circles of radii 4.em and 1 em touch each other externally and 6is the angle contained by:their direct common tangents. Then find sin 8. . Sol. Fig2.1 co = Ifangle C of triangle ABCis 90°, then prove that tan A +tan B=“ (where, a,b,care sides opposite to angles 4, B, C respectively) ab, Sol. Draw AABC with ZC=90°° b tan A+tan B= S42 bia Pp +b? ab ab @ Fig. 2.12 = tan™'3, then find the ratio BC:CA EF] inthe following diagram ZBAO: Sol. .. tan 6=3 oc oc te =tan 8, Be” cote wo FE A108 ota? 929 AC cord downloaded from jeemain.guru ‘Trigonometric Ratios and Identities 29 Trigonometric Functions of Any Angle Let 4 be a given angle with a specified initial ray. We introduce a rectangular coordinate system in the plane with the vertex of angle A as the origin and the initial ray of angle A as the positive ray of the x-axis (Fig. 2.14). We choose any point P on the terminal ray of angle A. Let the coordinates of P be (x,y) and its distance from the origin be r, then we define @ sin (i) cos A= = . (ii) tana => x iv) cot () seeA= = (vi) cose A= — y : 244 + These quantities are functions of angle A alone, They do not depend on the choice of point P and the terminal ray. If we choose a different point P” (x’,y") on the terminal ray of A at a distance r’ from the origin, it is clear that x’ and y’ will have the same sign as that of x and y, respectively, because of similar triangles AOPL and AOP'L’. Also, any trigonometrical function of an angle A is equal to the same trigonometrical function of any angle 360n-+ A, where n is any integer since all these angles will have the same terminal ray. For example, sin 60° = sin 420° = sin (-300°). After the coordinate system has been introduced, the plane is divided into four quadrants. An angle is said to be in that quadrant in which its terminal ray lies. For positive acute angles, this definition gives the same result as in case of a right-angled triangle since both x and y are positive for any point in the first quadrant. Consequently, they are the length of base and perpendicular of angle A. Graphs and Other Useful Data of Trigonometric Functions 1. y=fQx)=sinx Domain — R, Range — [-I, 1] Period> 2 sin?x , sinx| € (0, 1] sinx=O=>x= na, nef sinx= 1 =9x=(4n+ I)2,ne/ sinx=-1 9x5 (4n~)n/2,ne/ sinx=sina=x=nz+(-l)'ane/ sinx20=>x€ ()[2n, m+2nn] el downloaded from jeemain.guru 2.10 Trigonometry 2. y=F (x)= cosx Domain — R, Range —>[-1, 1] Period —» 27 cos*x , jcosx| € [0, 1] cosx=0=>x=(2n+ Iw2,ne 1 nm, ne T lax=(Qn+Innel cosx = cosa=9x= Inet cnet cosx20=>x€ Utne, 2ne+ 1/2] net 3: y=f(e)=tanx Domain > R~(2n+ Ia/2,ne 1 Range — (~29, 60) * Period > - Discontinuous atx=(2n+ 1) w2,ne I tan?x, |tanx] € [0,-) tanx=O=x"an,ne] tanx =tana=x=na+a,nel downloaded from jeemain.guru Trigonometric Ratios and Identities 2.11 4. p=f(x)=cotx Domain + R ~ nx, n€ 1; Range > (-~, ); Period —> 2, Discontinuous atx =n, n€ 7 cotx,, [eotx| © [0, 9) cotx=0=9x=(2n+1)n2,ne J : 5. y=f(x)=secx Domain — R~(2n+ 1)n/2, né J; Range — (-2,—1] U[I, ) Period + 2x, sec*x , |secx| € [1, e9) y Fig. 2.19 Domain > R~nr, ne I; Range — (-©9,—1] U[1, ©) Period > 2, cosec?x , |cosecx| € [1, c°) y Fig. 2.20 downloaded from jeemain.guru 242 Trigonometry Signs of the Trigonometric Ratios o1 r Functions ‘The signs of trigonometric functions depend on the quadrant in which the terminal side of the angle lies. We always take the length OP =r to be positive. Thus, sin -yir has the sign of yand cos = x/r has the sign of x. The sign of tan @ depends on the signs of x and y and-similarly the signs of other trigonometric ratios are determined by the signs of x and/or y. Sign can also be determined by the graphs. Thus, we have the following: [ Function Ist 2nd 3rd 4th quadrant quadrant quadrant quadrant sin 0 We We =ve ve cosec @ cos O We ve =ve We sec @ tan @ Fe ve we =ve cot t ut Variations in the Values of Trigonometric Functions in Different Quadrants 1 quadrant 2" quadrant 3™ quadrant 4" quadrant sin 0 1 from Oto 1 4 from 1100 4 from 010-1 T from -1 100 cos 0 from 1 to0 from 0to-1 T from—I to0 T from Oto | tan@ T from 010 2% T from—eoto 0 T from 0 to 0 T from 20 10 0 4 cot 0 J from to0 4 from 00-20 Lfcom 100 from 0 to -2 sec O T from | toe, F from—e 10-1 4-1 to-c0 from eto | cosee @ 4 from eto 1 T from | toes 1 from-to -1 + from-1 to-22 Note: Sunctions as well. Trigonometric Ratios of Standard Angles icreases in the interval (0, ‘+00 and eo are two symbols. These are not real numbers. When we say that (an @ increases from 0.10 © as @ varies from 0 to 4/2, it means that tan 8 in arbitrarily large positive values as 8 tends to x/2. Similarly, this happens for other trigonometricat m2) and it attains ‘Angle(@) > 30° 45° 60° T-Ratiol sin 8 Ww Wr 3/2 cos @ Vin ust 12 tan uJ 1 3 cosec 0 2 v2. 23 | sec 0 wi. v2 2 cot 3 1 uw downloaded from jeemain.guru Trigonometric Ratios and Identities 2.13, Transformation of the Graphs of Trigonometric Functions 1. Todraw the graph of y= f(x +a); (a> 0) from the graph of y=/(x), shift the graph of y=/lx), a units left along the x-axis. Consider the following illustration. Y yasnies) Fig. 2.21 ‘To draw the graph of y =x~a); (a>) from the graph ofy =x), shif the he graph ofy=x), aunits right along the x-axis. Consider the following illustration 2. To draw the graph of y = lx) + a; (a> 0) from the graph of y= f(x), shift the graph of y= f(x), a units upward along the y-axis, ‘To draw the graph of y = flx)—a; (a> 0) from the graph of y= /{x), shift the graph of y= ix), a units downward along the y-axis. 4 tnt Fig. 2.23, downloaded from jeemain.guru 244 3. Trigonometry Ify= fx) has period 7, then period of y= fax) is Tia}. ¥ y=sin (2x) Period of y= sin (x/2) is 2n0/(1/2)= 47 4, Since y= [{x)] 20, to draw the graph of y= [/la)), take the mirror of the gfaph of y= f(x) in x-axis for Ax) <0, retaining the graph for. f(x) > 0. Consider the following illustrations. downloaded from jeemain.guru Trigonometric Ratios and Identities 2.15 Here period of x)= |eos.x] is y=2se0x Fig. 2.29 Which of the following is possible? a sin @ 5 b tan O= 1002 2 «. cs 0=2*?- west) “a econ 1-9’ 2 Sol. b. sin @= 3 is not possible as—1 < sin@< 1. 2 2 + * a 1+ te ; P isnot possible, as in Tt ‘numerator is always greater than the denominator for —P “- cos 2 any value of p other than p = 0. Hence, a does not lie in [-1,'1]. a : downloaded from jeemain. guru 2.16 Trigonometry sec 0= ; is not possible as sec A¢ (-,—-1] ULI, -°). tan 0= 1002 is possible as tan @ can take any real value, Emin ‘Which of the following is greatest? ‘a. tant btand ¢.tan7 d tan 10 Sol. a, tan4 = tan(a+ (4—2)) = tan(4—n)=tan(0.86) tan7 = tan(2a-+ (7—2n))= tan(7 22) = tan(0.72) tan}0=tan(3+ (10-37) = tan(10 ~372) = tan(0.58) Now 1 >0.86>0,72>0.58 => tan] > tan (0.86) > tan(0.72) > tan(0.58) [as 1, 0.86, 0.72, 0.58 lie in the first quadrant and tan functions increase in all the quadrant] Hence, tan! is greatest. EEMENEM Which of the fottowing is least? a sin3 hsin2 sin 1 dsin7 Sol. d, sin 3=sin{x-(s—3)] =sin(—3) in x(n —2)] =sin(n—2 jin{27+ (7 -2n)] = sin(7-2n) = sin(0.72) Now 1:14>1>0.72>0.14 => sin(1.14)> sin] > sin(0.72)> sin(0.14) [as 1.14, 1, 0.72, 0.14 lie in the first quadrant and sine functions increase in the first quadrant] Hénce, sin 3 is least. Alternative solution: Fig. 2.30 From the graph, obviously sin 3 is least, sin8+ cos’@, then which of the following is not true? a. Maximum value ofA is - b Minimum value of 4 is—4 c. Maximum value of 4 occurs when sin@= 1/2 d_ Minimum value of A occurs when sin@=1 Sol. a,e,d. {(O~ 4sin@ + cos*@= 4sin@+ | —sin?@ = 5—(4—dsin@ + sin?) = 5 —(sind— 2)" Now maximum value of /(@) occurs when (sin@— 2)? is minimum. Minimum value of (sin@—2)" occurs when sin@= 1, then maximum value of f(8) is 5- (1 —2)? =4. Also minimum value off(6) occurs when (sin@—2)° is maximum. Maximum value of (sin@—2)" occurs when sin@=—1, then minimum value of f(8) is $—(-1 2)? =—4, downloaded from jeemain. guru Trigonometric Ratios and Identities 2.17 4x tsthequatonsec'0~ 7 * ; possible for real values of xand y? x+y xy (x+y)? Sol. Given, sec? day 21 (x+y)? Since sec?@ 21, we get => (ety)? sary => (xt+y)—4xy$O0or(x—y)’ 50 But for real values of x and y, (xy)? 20 Since (x-y)?=0, x=y.Alsox+y#0=>x#0,y#0 7 or is possible for real values of'x and y only when x= y (x #0). aty) : Therefore, the given equation sec? @= 1 Show thatthe equation sinO=x + — is impossible ifx is real. x i , 1 Sol. Given, sin 9=x+ — x 2 sin’o= x24 +2-(x-4) ava x, which is not possible since sin & <1. Ifsin?, + sin?@, +sin®@, = 0, then which of the following is not the possible value of cos, +c0s 0, + cos By. a3 h-3 e-l a2 Sol, a . / sin2@, + sin?@, + sin? = 0 => sin?@,=sin?@,=sin’@,=0 => cos7@,, cos*@,, cos’@,=1 => c0s6,, cos®, coSO, = + I 050, + cos®, + cos@can be-3 (when all are—I) or3 (when all are-+1) or-t (when any two are -1 and one +1) or! (when any two are +1 and one—1) but ~2 is not a possible value. EMMLELLM For real values of 6, which of the following is/are positive? a cos(cos @) b cos (sin'@) «sin (cos @) d sin (sin 8) Sol.a,b. cos @, sin @¢ [-1, 1] or (value lies in 1" or 4" quadrant) For which cos (sin ) is always greater than 0. sin (cos 8) <0, when cos @¢ [~1, 0} and sin(sin @)>0 when sin ¢ (0, 1} 1 EY Find the range of x)= =" ample Sol. I Scosx <1 =-4S4cosxs4 =-7S4cosx-3S1 downloaded from jeemain. guru 2.18 Trigonometry =-7S'4cosx-3 <0 or0<4cosx-3<1 (-° 4cosx-3¥0) 1 >-s00ree>——___ >} 4cosx-3 0 ~ Jeter Find th £62) = = — ind the range off(x) = <= — Sol. -I$sinx<1 =-SsSsinxs5 =-I1 $Ssinx-6s-1 1s — ee STS Ssinx—6 = -1,-1/11 Gaxag © Wn) Find the range of flx) = cos? x +secx Sol. We have Ax) = cos? x + sec? x (cos x ~ sce x)’ + 2 cos xsec x + (cos x—secx)' 22 Find the range of f(x) = sin? x~3 si Sol, fx) =sin’x—3 sinx +2 =(sinx-3/2)'+2-9/4 * sin x-3/2)"- 1/4 ~Issinxs1 =-5/2Ssinx-3/2< -12 = 1/45 (sinx-3/2)' <25/4 = 0S(sinx-3/2)!'-1/4< 6 Find the range‘offix)= ysin?x- 6 sin x49 +3. Sol. fx) = yin? x—Gsinn+9 +3 = Visine = 3)? +3 =|sinx—3|4+3 Now-I Ssinxs! =>-436 WCE Find the value of x for which f(x) = ,/sin x - cos x is defined, x€ [0,2n}. Sol. fix)= sin x—cos x is defined if sin x2 cos x, From the graph, sin x 2 cos x, for x € [5 =| Which of the following is highest? a, cosee 1 b cosee 2 +e cosee 4 cose (6) Sol. d. Consider sin 1, sin 2 and ~sin 6 (sin 4 is negative; hence, cosec 4 cannot be maximum). Fig, 2.32 From the graph, sin (~6) is least, hence cosec (-6) is maximum. Solve tan.x>cotx, where x € [0,2]. downloaded from jeemain.guru 2.20 Trigonometry Sol. a Fig. 2.33 We find that tan x 2 cotx ‘Therefore, values of tan x are more than the values of cot x. That is, values of x for which graph of y= tan x is above the graph of y = cot x. From the graph, itis clear thatx ¢’(1/4, 2/2) UGw/4, m)U (Sx/4, 3012) UCT H4, 20). 1, Find the least value of 2 sin? @+ 3 cos? 6. 2, Find the range of f(x) = sin (cos x). 3. Find the range of 12 sin @—9 sin? 0, 4, Find the minimum value of 9 tan? +4 cot? @. 5, Which of following is correct (where ne N)? 2 a sino="*1 hy sing 21g seco? a seco= —2 7m nal ni In 6. If'sin? 0, + sin? @, + +sin? 6,=0, then find the minimum value of cos 6, + cos 8, + 7. If sin? @= x? — 3x +3 is meaningful, then find the values of x. +c0s 6,, 8, Find the range of fix) = downloaded from jeemain. guru Trigonometric Ratios and Identities 2.21 9. Find the range of 2) = 5 10. Find the range of flx) = cos*x + sin’x— 1 11. Find the minimum value of the function x) = (1 +sinx)(1 +cosx), Wxe R. 12. Prove that (sin 8-+ cosec 6)° + (cos 0+ sec 6)? 29. PROBLEMS BASED ON TRIGONOMETRIC IDENTITIES Show that 2(sin‘x + cos®.x) ~3(6it Sol. 2(sin°x + cos® x) — 3(sin* x + cos*r) +1 = 2[(sin2x)? + (cos* x)°] — 3(sin* x + cos*x) + 1 = 2[(sin?x + cos? x) — 3sin?xcos*x(sin?x + cos*x)] —3{(sin? x + cos*x)?—2 sin? xcos?x] + | = 2[1 — 3sin?xcos?x] - 3[1 ~2 sin? xcos’x] +1=0 prvothit [82 = sec ateno, Zeoee, 1-sin@ 2 2 l+sin 8 H+sin@ 1+sin8 Sol. .L.H.S.= ~ 7 in 1=sin@ 1+sin8 = |itsiney? _ |(1+sina)? 1=sin? @ cos? 0 + cos'x) +1=0. 1+sin@ sin@ = =— = sec@ +tan cos cos cos = RHS, 1 1 1 Prove that —~———- eae secA-tanA cosA COSA sec A +tanA~ 1 1 1 1 LT = Sol. To prove (sin? 8+ cos?@) + 25 (sin?@+ cos*6) = 25 +2?:. ‘ => 34525427 or? =9 lai downloaded front jeemain.guru 2.22 Trigonometry = x02) If (sec A + tan A) (see B+ tan B) (secC+ tan C) =(secA ~ tan A) (sec B — tan 8) (secC—tan), ° prove that the value of each side is + 1. Sol. Let (secd + tan A)(sec B+ tan B) (sec C+ tan C) =x 0 (sec A tan A) (sec B—tan B) (sec C-tan C)=x. Gi) Multiplying Eqs. (i) and (ii), we get | “ (sec?A —tan2A)(sec?B — tan2B)(sec*C — tan*C) = x? ores] fxn! Hence, each side is equal to + 1 Iftand+ secO~ 1.5, find sin@, tan8, and sec0. Sol. Given, 2200+ tand= 3 i . 0 Now, sec6-tand= = 2. (see Fig, 2,34) “ (i ow, se¢0—tan= = (see Fig 2, Adding Eqs. (i) and (ii), we get 2500 = % 18 13, = 13 see 12 B (Ne 5 12 tan = as 12 ig. 2. 1 i = cos = = cosO= Again see 8—cosO= n, or = n ee I=cos? @ or Sit i cos 088 2 From Eq. (),sin = 2. m cos‘ 6 Putting in Eq. (ii), we get — =n, or cos? @ = mn mn? cos os = (nx), or eos = (nn) «) downloaded from jeemain.guru Trigonometric Ratios and Identities 2.23 2 2 24) 1 aa a _ (nn + From Eq. (iii), sin @= °° 9 = (mtn) = = (mn?) ™m 2 sin?0= (mn?) o Adding Eqs. (iv) and (v),, we get 2 2 (in?n)? + (mn?) = cos? + sin? @ 2 2 or. (nn) + om?) = ‘ ‘ a if med + oe = 1, then prove that cos'B sin’B ‘ 7 a. sind + sin’ =2 sin? sin?B » See = cos*A sin’A 4 ‘ , A Sol. Given, 4 <1 (cos? A+ sin? A) fA ‘ in’ A = cos? A= sin? a - cos” B sin? B cos? A(cos? A ~cos* 8) (sin? a -sin® ) = sin? a cos? B r sin? B = Acosta costa) = S04 [1 ~cost2) -(1 cos] sin? B a cos? A z = : A(cos? A-cos*B) ~ cos! sin’ B (cos? A ~cos? a cos? A _ sin? A) _5 cos’ B sin? B => (cos’A—cos*B) ( when cos"d ~cos*B = 0, cos? = cos*B 0 2 cos’ A ns when 2 = 0, cos” Asin? B = sin? Acos? B cos* B => cos*d(1 ~ cos*B) = (1 — cos?4) cos*B => cos" ~ cos? cos? B= cos*B — cos cos?B = cos’4 =cos*B ii) Thus, in. both the cases, cos?4 = cos?B. downloaded from jeemain.guru 2.24 Trigonometry 1 -sin?d = 1 —sin?B, or sin®A = sin?B in’ + sin’B = (sin? —sin?B)* +2 sin2A sin?B = 2 sin’ sin? B=R.HS. [+> sin? =sin?B] a LHS as ait Ae ent cos'B , sin’ B _ cos’ 8, sin‘ B bh LHS.= me cos*A sin? cos*B sin? B = cos*B + sin’B= 1 Ifx=sec 8—tan Oandy=cosec 0+ cot 8, then prove that ay +1=y—x. sin@ }+cos® a l-sin8+cos@ cosd sind sind cos Sol. xy+1 (sin? 0 +cos? @) _(sin@-cos6) sinOcosd sin cos@ = (tan + cot 6) ~ (sec 8~ cosee 6) = (cosec 0+ cot 6)—(sec @— tan 8) = y—x Concept Application Exer 3 Show that 3(sin x—cos x)* + 6(sin x + cos x) + 4(sin® x + cos® x) = 13. If sec + tan @= p, then find the value of tan 6. If (1 +sin A) (1 + sin B) (J + sin C)= (1 —sin A) (1 ~sin B) (1 —sin C), then find the value of (I + sin A) (i +sin B) (1 +sin C). If (see 8+ tan 8) (sec $+ tan ¢) (sec y+ tan y) tan Ban ptan y, then (sec 9—tan 6) (sec tan ¢) (sec y—tan yA is equal to wen - a. cot Bcot dcot y b tan Btan tan y -€. tan @+ tan @+tan yd cot + cot + cot y a . If sind=1, ~sin@->cos@=1, then eliminate 8. a b ab Ifa+ b tan = sec Band b—atan = 3 sec 6, then find the value of a? + b If a sin? x + 6 cos? x = c¢, b sin? y + @ cos? y = d and a tan x = } tan y, then prove that ne TRIGONOMETRIC RATIOS FOR COMPLEMENTARY AND SUPPLEMENTARY ANGLES In each of the following figures, x and y are positive. Also triangles OPM, OP’M’, or OP’M are congruent. downloaded from jeemain.guru Trigonometric Ratios and Identilies 2.25 sin(-@) =-sin® sin (-€¢)=— r cos(-6) = cosO cos (-8) = 4 = cos 6, tan(~6)=->=tan@ Taking the reciprocals of these trigonometric ratios, we have cosec(—8) = = cosec 6, sec(~6) = sec @and cot(~A) =~cot 8 i ; e a sin(90°~ 6)= cos sin(90—6)= = cos 60s(90° — 8) = sind 0s(90~ 6)= X=sin8 r tan(90~ 6) =~ =cot @ # | : sin(90° + 6) = cos8 sin(90 + 6) = ~=cos0 r cos(90°+ 6) =—sind t——-++ sin (180°—6)=sind cos (180°) =—cos0 sin (180° + 8) =sin(180+ @) =-sin 6 cos(180+ = cos(180° 8) tan(180 + @)= =—00s 0 L downloaded from jeemain. guru 2.26 ‘Tigonometry Since the terminal sides of co-terminal angles coincide, hence their trigonometrical ratios are same. Clearly, 360° — 8 and—@are coterminal angles. Therefore, sin(360° ~ 6) = sin(-8) = -sin 6, cos(360° ~ 8) = cos(—6) = cos 6, and tan(360° — 8) = tan(—@) = -tan 8. Similarly, cosec(360° — @) = ~cosec 6, sec(360° — 8) = sec Band cot(360° - @) =—cot 8. Also @and 360° + Bare co-terminal angles. Therefore, sin(360° + @)= sin @, cos(360° + 8) = cos 6, tan(360° + 0)=tan @, sec(360° + 8) = sec 8, cosec(360° + 8) = cosec Band cot(360° + 6) = cot 4. In fact, for any positive integer n, (360° x n+ 6) is co-terminal to 8. Therefore, for any positive integer n, we have sin(360° x n-+ 8) =sin 8, cos (360° x n+ 8) = cos 8, tan(360° x n+ 8)=tan 8, cosec (360° x n+ O)= cosec 8, sec(360° xn + 0) =sec@and cor(360°xn+@)=cot 0. * Prove that sin (420°) (cos 390°) + cos (-660°) (sin 330°) Sol. L.H.S.=sin(-420°) (cos 390°) + cos (660°) (sin 330°) sin 420° cos 390° + cos 660° sin 330° [ v sin (@)=—sin 0, cos(—0) = cos 6] sin (90° x 4 + 60°) cos (90° x4 +30°)+ €0s (90° x7 + 30°) sin(90° x3 + 60°) (sin 60°) (cos 30°) + (sin 30°) (cos 60°) 8 B44) 22 "2h 2; =RHS. €05(90° + 8)sec(-0)tan(180° ~ 0) Example 2.42 SS Prove that (360° —O)sin(180° + 0)eot(90° 0) cos(90° + 8)sec(-6)tan(180°~4) __ (~sin@)(sec@(~tan®) sec(360° —@)sin(180° + A)cot(90°- 6) (sec @)(—sin @(tan 8) If A, B, C, D are angles of a cyclic quadrilateral, then prove that cos A + cos B+ cos C+ cos D=0, Sol. We know that the opposite angles of a cyclic quadrilateral are supplementary, i. A+C= @ and B+ D= x. A=n-CandB=2-D => cos A=cos(#—C)=-cos C and cos B= cos (7D) =~—cos D cos A + cos B + cos C+ cos D=—cos C—cos D + cos C+ cos D= 0 Show that tan 1° tan2° .,, tan 89 Sol. L.H.S.= (tan]° tan89°) (tan2° tan88°) ... (tan44° tan46°) tan45S° = [tan] °tan(90% 1°)] [tan2° tan(90°-2°)] .. [tan 44° tan(90°— 44°)tan45? = (tani? cotl®) (tan2° cot2°) ... (tan44? cotda°ytan4s? = 1 [+ tan @cot @= 1 and tan45° = 1] Show that sin?S? + sin? 10° + sin? 15°+---+sin?90°= ot Sol. L.H.S. = (sin®5° + sin°85°) + (sin?10° + sin?80°) + -~ + (sin240° + sin?50°) + sin?45°+ sin?90° = (sin?S° + cos?5*) + (sin?10° + cos?10°) + --- + (sin?40° + cos*40°) + sin?45° + sin290" Sol. L.H.S= 1=RHS. 2 1 1 mastetsrersiersy+(4) +1294 v2 e downloaded from jeemain. guru Trigonometric Ratios and Identities 2.27 3n Sn 7m Find the value of cos* + cos? +cos? = +cos? 16 16 16° 46 7 xn Sol. LHS.= cos? 4-005? 2% 4.005?( % — 3% eos'( -*) 16 16 2 16 2 16. a wale 23m 23m 22 = cos? tc00? SE + sind 2% sin? Z cos? TE teas! 7 + int sin? (oe £2) low at) =l+l=2 Ifsin (120°—a) =sin (120° f), 0 A=Bord=n2-Bic,A+B=H4 => 120° a= 120°~ f, or 120°- a+ 120°— B= 180° = a= Borat B=60° 1 Concept Application Exercise 2.4 1. In triangle ABC prove that a sind =sin(B+C) b sin24=—sin(2B +20) c. cos =—cos(d +B) A+B é ‘d tan] —— }=cot— ()-=$ . Prove that sin (~-420°) (cos 390°) + cos(=660°) (sin 330°)=—1 3. Prove that a. tan 720° —cos 270° —sin 150° cos 120°= ; ‘b sin 780° sin 480° + cos 120° sin 150°= — Alfa= 7 s Prove that cos a cos 2arc0s 3acos 4arc0s Sacos 6a= -2. 5. Find the value of tan tan 2 tan = tan 2 tan 2 i 20°: 200=—« 20 20 «20 coi 54° tan 20° fs tan36° cot 70° 6. Find the value of 9k ok Wt 7, Prove that sin? tsi? Z ssin? 4: 9 1B 9 a rove (26-3) 0) mo 9. In any quadrilateral ABCD, prove that a sin (A+B) +sin(C+D)=0 th cos(4 + B)= cos (C+ D) downloaded from jeemain.guru 2.28 Trigonometry TRIGONOMETRIC RATIOS FOR COMPOUND ANGLES Cosine of the Difference and Sum of Two Angles 1. cos (A—B) = cos A cos B+ sin A sin B 2. cos (A + B)=cos A cos B-sin A sin B forall angles A and B. Proof: 1. cos (A-B) Let X’OX and YOY’ be the coordinate axes, Consider a unit circle with O as the centre (Fig. 2.35). Y Py(cos A, sin A) Ps [cos (A~B), sin (A—B)] Fig. 2.35 Let P, P, and P be the three points on the circle such that ZXOP, = 4, ZXOP, = Band LYOP3=4-B. As we know that the terminal side of any angle intersects the circle with centre at O and unit radius at a point whose coordinates are the cosine and sine of the angle, Therefore, coordinates of P,, P and Ps are (cos A, sin A), (cos B, sin B) and (cos (A —B), sin (A —B)), respectively. We know that equal chords of a circle make equal angles at its centre and chords Pg ?3 and P,P, subtend equal angles at O. Therefore, : Chord PyP;= Chord P,P, = yleos(A—B)=1)? + {sin(A— B)—0)? = y(cos B—cos A)’ + (sin B-sin AY* = {c0s(4— B)~ 1}? + sin’(4 ~ B)= (cos B—cos A) + (sin B—sin Ay => c0s(A - B)—2 cos (A - B) + | + sin?(A - B)= cos? B + cos’ A -2 cos A cos B+ sin? B +sin?A-2sind sinB = 2-2co0s(A-B)*2-2cosA cos B—2 sin A sin B = cos(4~B)=cos A cos B+ sin A sin B 2. cos(A +B) = cos (4 -(-B)) @ = cos A cos (~B)+ sin 4 sin(~B) [Using Eq. (i)] = cos A cos B-sin A sin B [-> cos (-B) = cos B, sin (~B) =-sin B Hence, cos (A + B) = cos A cos B-sin A sin B This method of proof of the above formula is true for all values of angles A and B whether positive, } Note: ' | zero or negative, downloaded from jeemain.guru ‘Trigonometric Ratios and Identities 2.29 Sine of the Difference and Sum of Two Angles if 2 Proof: 2. sin (A~B)= sin A cos B—cos A sin B sin (A+ B)=sin A cos B+cos A sin B We have sin (A—B)=cos(90° (4 -B)) [+ cos (90° — 8) = sin 6} = cos ((90°—A) +B) = cos (90° - 4) cos B- sin (90°— A) sin B =sind cosB—cosd sinB sin (A+ B)=sin(4—(-B)) 0 = sin 4 cos (-B) — cos A sin (-B) [Using equation (i)] =sin A cos B+ cos 4 sin B [> sin(@-8)=-sin B] Tangent of the Difference and Sum of Two Angles » . tan (A+ B)= . tan (A —B)= tan A+tan B tan Atan B tan A-tan B T+tan Atan B |. We have in(A+ B) _ sin Acos B+cos Asin B cos(A+B) cos Acos B—sin Asin B _, fan A +tan B tan AtanB tan(4~B) = tan (4 +(-8)) tan (4+B)= ) [On dividing the numerator and denominator by cos A cos B] _ tan A +tan(-B) © T= tan Atan(~B) = tan A-tanB I+tan AtanB Similarly, it can be proved that cot Acot B-1 [Using Eq. ()} ‘cot (4+B)= cot B+ cot A cot Acot B+1 and cot (A — B)= SAAC +1 a8) cot B—cotA Some More Results Te 2 3. 4, & sin(A + B) sin (A ~B) = sin? A - sin? B= cos* B—cos*A cos(4 +B) cos(4 ~ B) = cos? A —sin? B= cos? B— sin? A sin(A + B+ C)= sin A ¢os B cos C+ cos A sin B cos C+ cos 4 cos B sin Csin A sin B sin C cos(d + B+ C)= cos A cos B cos C— cos A sin B sin C-sin A cos B sin C—sin A sin B cos C tan A+tan B +tan C—tan Atan BtanC n Bian C—tanC tan A tan(4+B+C)= mt OF oan Atan B downloaded from jeemain. guru 2.30 Trigonometry Proof: 1. sin(A +B) sin(d — B) = (sin A cos B + cos A sin B) (sin A cos B—cos A sin B) ? A cos? B- cos? A sin® B sin? A (1 —sin? B)-(1 - sin? A) sin? B = sin? A—sin? A sin? B sin? B +-sin? A sin? B= sin? 4 sin? B = (1 -cos® A)- (1 — cos? B) = cos* B- cos? A 2. cos (A +B) cos (A ~B)= (cos A cos B—sin A sin B) (cos A cos B+ sin A sin B) . = cos? d cos’ B~ sin’ A sin? B = cos? A (I —sin? B) -(} —cos? A) sin? B= cos?A - sin? B =(1-sin? 4) —(1 ~ cos? B) = cos? B= sin? A $. tan(4 + B+C)=tan(A+B)+C) tan A+ianB tan(A+B)+tanC _ _1=tanAtanB tan A+ tan B+tanC ~ tan Atan BtanC * Ttan(a+ Bytan€ j_(tanA+tuanB) | tan Atan B—tan Bian — tan Cian A tan Alan B. sin(B - C) 4 sin(C -A) +s sin(A-B) cosB cosC cosC cos cos A cosB im Prove that Sol. Firstterm of L.H.S. is BcosC _cosB sinC sin(B-C) _ sin B cosC — cos. sin€ — —— = tan B-tanC cos B cosC cos BeosC cos BcosC cos BcosC Similarly, second term of L.H.S. = tanCtand and, third term of L.H.S. =tand ~tanB Now L.H.S. = (tan B=tan C) + (tanC ~tan A) + (tand ~tanB)=0. Ifsin sin B—cos acos B+ 1 =0, then prove that 1 + cot atan B=0. Sol. Given, sin asin B~cos cos B+1=0 or cos acos B-sin asin B=1 orcos (a+ f)=1 @ Now I +cot ortan B= 1+ S82 5, S108 sina” cos _ Sin @ cos B + cosarsin B sinarcosB (a+ B) sina cos B —cos* (a+ B)=1—1=0) [ (a+ p= inet cos Show that cos?@+ cos*(a+ @)-2.cas cos Ocos (a+ 6) is independent of 6. Sol. 030+ cos’(a+ 8) ~2 cos cos Ocos (A+ 6) = cos*8+ cos(a+ B)[cos(a+ 8) ~2 cos acos 6] = cos? + cos(a+ 8)[cos ccos Asin arsin 8-2 cos acos 8 downloaded from jeemain. guru Trigonometric Ratios and tdentitios 2.31 =cos?6—cos(a+ [cos acos @+ sin asin 6] c0s20—cos(a+ 6) cos(a— 8) 0s'8— 5 [cost ‘a sin®6] = cos?@ + sin2@— cos?@ —cos*er, which is independent of 8. If3tan Otan g=1, then prove that 2 cos (0+ 9) = cos(0- 9). Sol. Given, 3 1an @tan 9= Lor cot Ocot p= 3 cos cosp _3 sind sing 1 By componendo and dividendo, we get cos@ cos +sin@ sing _3+1 cos@ cosp—sindsinp 3-1 cos(0- 9) _ cos (09) = aoe) sola =, 00s (A+B Gee ifsin(4-2)= Sr ped cya in ra ml Mao Oand m4. ii 7 _py = tan (A+B) +tan (A-B) Sol, tandA=tan|(A+B)+ (4-8) = Ta ay nA 8 0 Given that,0 tan70?—tanS0° tan 20° tan70° = tanS0° + tan20° => tan70° = tan70° tan50° tan20°%+ tan50° + tan20° = tan(90° — 20°) tan50° tan20° + tanS0° + tan 20° = cot 20° tan 50° tan 20° + tan 50? + tan 20° = tain 50° + tan 50° + tan 20° = 2tan $0°+ tan 20° Let A, B,C be three angles such that A + B+ C= a. Iftan A- tan B=2. Then find the value cos A cos B cosC of Sol. Given tan A: tanB=2 cosAcosB __ cos A-cosB cos A-cos B cos cos(A+B) sinAsinB—cosA cosB Let y= 1 1 “tan AtanB-1 2= Range of f(@) = acos@ + bsin@ Let a= sincrand b= cosa, then 7? = a? + 6? and tance= alb Now f(@) = acos0 + b sin@=r sin ccos6+r cos crsin 0 =r sin(8+ a) = Va? +b" sin( 9a <) Now-1s sin( tan" ) <1 = Va? +0? s Pe sin oan 2) lato ' Hence, range is [We +b, Vet +e. downloaded from jeemain. guru ‘Trigonometric Ratios and Identities 2.33 Find the maximum value of V3 sin.x+ cos x and.x for which a maximum value occurs, Sol. V3 sin x+cosx= B nse deins|e2 sin (x-+ 1/6) qmatgene which is maximum when x + 1/6 = a/2 or x = 60° and has a maximum value 2. Find the maximum and minimum values of cos? 0-6 sin 8 cos 0+3 sin? 0+2. Sol. cos? 0~6 sin Ocos 6 +3 sin? 02 _ [+008 26 = 00826) |» ~3sin20434 = 4—cos26—3sin 20 Now, ~c0s26-3sin 20 [-/10, vi0] => 4-cos20-3sin29€ [4-v10, 4+vi0] Concept Application Exercise 2.5 . IfA+B= 225°, then find the value of 2A, 2018 Toot A” I-+cotB . IFtan A ~tan B= x and cot B— cot A =y, then find the value of cot(d ~B). . fx is A.M, of an 2/9 and tan Sa/18 and y is A.M. of tan 2/9 and tan 77/18, then relate.x andy. 2 2 Prove that “22 28= "20. tan 30tan 0, 1=tan? 26tan? 6 5°, show that (1 + tan A) (1+ tan B)=2, /2, tan B= 1/3, then prove that cos 24 = sin 2B, . Find the maximum value of 1+sin(% +0) +2sin & -0) on all real values of 8. . Find the maximum and minimum values of 6 sin x cos x +4 cos 2x. HF pbx) = sin x (Bin? +3) + 008 x (cos) x +4) + ; sin? 2x-+ 5, then find the range of p(x). ). Find the value of cos sin 4.cos™ ra 12 8S . If.cos(ar+ ) + sin(ar— B) = 0 and tan BI, then find the value of tan a . If sin A + cos 24 = 1/2 and cos A + sin 24 = 1/3, then find the value of sin 3.4. . Ifsin. x + sin y+ sin z= 0=cosx+ cosy + cosz, then find the value of expression cos(@— x) + cos(9—y) + cos(0—z). downloaded from jeemain. guru 2.34 Trigonometry TRANSFORMATION FORMULAE Formulae to Transform the Product into Sum or Difference ‘We know that sin A cos B+ cos A sin B=sin (A+B) sind cos B—cos A sin B= sin (4 ~B) cos A cos B—sin A sin B= cos (4 + BY 0s A cos B + sin A sin B= cos (A~B) Adding Eqs. (i) and (ii), we obtain 2 sin A cos B= sin (A+ B) + sin(A~B) ‘ w Subtracting Eqs. (ii) from (i), we get 2.cos A sin B= sini(A + B)—sin (4B) (wi) ‘Adding Eqs. (ii) and (iv), we get 2 cos A cos B= cos (A + B)+.cos (A - B) : (vii) Subtracting Eqs. (iii) from (iv), we get 2sin A sin B= cos (4 B)-cos (A +B) Formulae to Transform the Sum or Difference into Product and f= O22. 2 LetA+B=Candd—B=D. Then, A= £2? Substituting the values of 4, B,C, and D in Eqs. (v), (vi), (vii), (vii), we get sin C-+sin D= asin( $2? oe $=?) @ (ee: ® 2a) 0 2)u(e=) (2) a 2)o(22) These four formulae are used to convert the sum or difference of two sines or two cosines into the product of sines and cosines. sine-sindD=2sin( © cos C+cos D= 2s (SE cos D-cos C= 2in( or csC-cos D= p= -2sn(S2)s or cos C-cos D=2 sin (+ T Ifsin A =sin Band cos A= cos B, then prove that sin AA Sol. We have sin A = sin B and cos A = cos B = sinA~sin B=0andcos A-cos B=0 downloaded from jeemain. guru Trigonometric Ratios and Identities 2.35 =P )so( 422) =0 dine tekst = 20053 + F yg SHO 5 cosi75° = 2.cos 60° cos (-5%) + cos 175°=2% $ c0ss° + cos (180°- 5°) =cos $°—cos 5°=0 Prove that cos 18°—sin 18°= V2 sin 27°. Sol. L.H.S.=cos 18°—sin 18°= cos 18°— sin (90° 72°) =00s 18%-cos 72° wm 25in BAT gin, 22° = 18° yo = 2sin 45° sin27°=2—~ sin27°= J2 sin27° ERM Provethat sinSA -sin3A sinA +sin3A ee Stan A Sm assnee = tan2A ® cosSA+cos3d bcos A +c0s3A Sol. SAW3A) (SA+3A 2 JO 2) | 2sin Acos4a _ a LHS,= 22S a = an A=RHS, COSSA+COS3A 9.56! a8) (4 zA) 2cos4AcosA 3A+A) (3A-A el a b LHS. ~ paso) eens = tan2A=RHS. 2 Prove that cos a-+c0s cos y+ eos (a+ B+ ))= cos 2Ag os FEL cog lA, Sol. L.H.S.=cos a+ cos B+ cos 7+ cos (a+ B+7) = (cos a+ cos B) + [cos 7+ cos (r+ B+ 7] ll tt of bt 8 = om 2) 1 0 S 2 a R nv pra downloaded from jeemain.guru 2.36 Trigonometry =o) 2) am Pl sin A + sin2A+ sin4A + sinSA Provertag tA tein sind + sin cos A+ cos2A+e0s4A+eossa "34 sin A+sin2A+ sin4A + sinSA cos A+cos 2A +cos 4A +cosSA Sol. (sinSA+ sin A} +(sin4A+sin2A) (cosSA + cos A) +(cos4A+cos 2A) "i _2.8iN3A cos2A + 2sin3Acos A * Zeos3Acos 2A +2c083Acos A _ 2sin3A(cos2A + cos A) _ 2cos3A(cos2A+cosA) z or 0, accordingly as n is sin (— 6) =—sin 0] cot? A= 0, if nis odd A~B ifniseven, 2cot” Prove that (cos a+ cos fy? + (sin a+ sin By?=4cos* (3). Sol. L.H.S,=(cos a+ cos )? + (sin oc+sin B)? a wen 222 ao 22) + {ose( 228) 258) 4 “ scos8( Poot BB aint downloaded from jeemain. guru Trigonometric Ratios and identities 2.37 = 4cos (2) [cot SEB aint 92 2 2 2 =RHS, Msec (8+ a) + sec (~ a) =2 sec 6, then show that cos? @= I + cos a. Sol. sec (+ @) + sec (@- a) =2 sec @ 1 1 2 3 ——++_.-4. cos(@+a) cos(8—a) cos 5 CoO= 0) +c0sO+a) 2 cos(8+a)cos(8—a) cos 2cosBcosa __2 a SC cos? @=sin? qq cos => cos?@ cosa= cos?6— sinter => sin’a@=cos?@(1 —cosa) => 1-cos*a=cos*@(1-cosa@) = 1+cosa=cos?@ In quadrilateral ABCD if sin (4 a ) on(4 : 5) +sin ( 7 A B Cc D t tA ae ate tn then find the value of sin sin sin sina Sol. sin( AZ Jeo 45*) +s( $5?) m( S52) 2 a 2 2 = Flsind +sin B +sin C+ sin D)=2 2) (¢ -D cos| 2 =sinAtsinB+sinC+sinD=4 2A=B=C=D=90° asin 4 sin sin’ sin? = 4 Concept Application Exercise 2.6 1. a. Prove that sin 65° + cos 65° = /2 cos 20°, b. Prove that sin 47° + cos 77° = cos 17°. 2. Prove that cos 80° + cos 40° — cos 20° 3. Prove that sin 10° + sin 20° + sin 40° + 0. 50° = sin 70° + sin 80°. on 1n 4, Prove that cos “+008 4-005 +605 =0, 5 5 5 5 ‘ 1 4 2 S Wainer an Be and ton f= con em ctiowitiatont 2, 3 2 2 3 A+B 6. Ifeosee A + see A= cosee B+ sec B, prove that tan A tan B= cot. 7. Prove that sin 25° cos 115°= Fisinaor— ). downloaded from jeemain. guru 2.98 Trigonometry 8, Ifcos A 3 then find the value of 32 sn( sn (2) © 9. Ifxcos @= y cos (o-%) =z 00s (0+) prove that xp+yz +x=0, 10. Ify sin p=x sin(20+ @), show that (x+y) cot (0+ @)=(»—x) cot 8. 11. If cos (A + B) sin (C+ D) = cos (A ~ B) sin (C - D), prove that cotd cot B cot C= cot D. 12. Iftan(d + B)=3 tan A, prove that . a. sin(24+B)=2sinB bh sin2(4 + B)+sin24=2sin2B TRIGONOMETRIC RATIOS OF MULTIPLES AND SUB-MULTIPLE ANGLES Formulae for Multiple Angles 1. cos2A = cos(4 +4) = cos" —sin?d © 1 —2sin?A = 2cos"A—1 Also sin? A= 4Q-cos2A), cosa - Hllteos 2A) 2, sin2A=sin(A+ 4)=sind cosd + sind cosd = 2sind cosd tanAttanA _ 2tanA tanA tan A j—tan? A 3. tan24 = tan(A +4): 4, sin34 = sin(24+4) . in 24 cos A + cos 24 sin A=2 sin A cos. A cos A +(1 2 sin? A) sin A sin A cos? A+ sinA~2 sin? A sin A (1 -sin? A) + sinA-2 sin? A =2sinA-2sin' A+ sind —2 sin’ A =3sind-4sin'A 5. cos3d=cos(24 +A) = cos 24 cos A—sin 24 sin A= (2.cos*A~ 1) cos 4-2 sin A cos sin A =2 cos? A—cos A ~2 cos A (1 —cos? A) eos? A—cos A-2 cos A +2.c0s) A cos? 4-3 cos A 6. sin 2A and cos 24 in terms of tan A 2sinAcosA __2tanA cos*A+sin?A I+tan7A sin24=2sin A cos A= [dividing numerator and denominator by cos? 4] cos? A=sin?A _1-tan? A 082d = cos* A—sin? A= = cos*A+sin° A 1+tan* A [dividing numerator and denominator by cos? A} 1-cos2A i+cos2A Also tan? = downloaded from jeemain. guru Trigonometric Ratios and Identities 2.39 7. Inthe formula oftan(4 +B + C), putting B= A and C= A, we get ast tangam 3tanAtan? A 1=3tan? A 34 Similarly, we can prove that eot 34 = £0 A= 3e0t A 3eot? A=I Sy = Sy 4S. 8. tan (A, +A) + 4,) = LS an (Ay +p = dy tS where 5, = tan A, + tan A, + --- + tan 4,,= Sum of the tangents of the separate angles, S, = tan A, tan 4, + tan A, tan A, + --- = Sum of the product of tangents taken two at a time, S,=tan Ay tan A, tan A, + tan A, tan A; tan A, + --- = Sum of the product of tangents taken three at atime, and 0 on, Af A\=Ay= + =4,=4, then we have S, = ntan A, S)="C> tan? A, S;="C, tan? A, --- Provetia . sin20 sin20 ——, =tand to ® T+e0s20 b Treos20 1+sin20+cos20 1+sin@-cos@ —————- = ot 0 —————_ = tan © Tesin20—c0s20 © Tesind+cosd 0820 tan (ai4—0) ¢ ee i228 1+sin20 A+sind 42 Sol. a Lins.= S228 _ 2806080 ong = RMS. 2cos* @ = 2806086 «cog = RUS, 2sin? _ 1+sin20+c0s2@ _ (1+c0s26)+sin 26 Ls = 4 . T+sin20—cos20 (1-cos26)+sin 20 = 20087 8+2sinOcosd 2sin® @+2sin cos _ 2cos0(cos8 + sin) 2sin@(cosO+sind) sind 4 Lins, = Ltsind=cos@ _ (1-cos6) + sind I+sinO-+cos@ (1+cos@) +sin asin? 2 +28in ooo 8 din ${sin8 +058) 208 aun Se® aeo8[sndscu) = uno -RHS. 2 3 downloaded from jeemain. guru 2.40 Trigonometry e LHS.= 2 L+sin29 1 i it ————— . Provethat 1 a3 (is) __ sin? @+cos?0-+2sincosO sin? @+c0s” @—2sin 8cos 0 . 2 2 a (satus) te ( sand) (dividing numerator and denominator by cos 6) sind—cosd) — \i-tand RECESON 1f + 6=90°, find the maximum value of sin asin B. Sol. Lety=sin arsin B= sin arsin (90°— 0) sin acos a= Fsin2o0 which has the maximuri value 1/2 when sin 20= 1, 1-tan?(4-4) Prove that ———--—~<-= sin2.A. 1+tan?(-a) . 4 1-tan?(Z-a) ag sou’ ——t_ . (vere a= 0s 20= oos(£-24) 7 vstan?(Z-a) 1+ten"@ -B Prove that (cos A ~ cos B)?+ (sin Asin B)?=4 sin? A= 2 Sol. L.H.S.= (cos A~—cos B)’ + (sin A~sin'BY = cos? A + cos? B-2cos A cos B+ sin’ A + sin® B-2sin A sin B downloaded from jeemain.guru ‘Trigonometric Ratios and Identities 2.01 = (cos? 4 + sin? A) + (cos? B + sin? B)— 2(cos A cos B+ sin A sin B) = 2~2cos(4 —B)=2( 1 —cos(4- B))=4 sin? Ifsin A= 3 and 0° 0as 00 x 0<2B<=— > B 3 Hence,0 Sot. Wehave % = 2% ang = _— 3% 8 3 8 8 In x Sx 3x = cos =~cos™ and cos >= =-cos = 8 8 8 8 50 _ a3 ain = cost 2 =cos*™ and cos = = cos & 8 8 LHS.= 2cos* 2 420058 downloaded from jeemain. guru 244 ‘Trigonometry’ 2 2 3n =2 of Z) {os 22) ( 8 8 2 a 140s) [140082 eae =2 4\ joa - 00820 60s? 2 Zz 2 2 “3fa) a} -Hfiede Vea) im nus Mtmex- | + sin] 2 cos? x x {cos | — [sin] Baad —cos = +sin= 2.2 nexcdn, 2c Xen ~ cos 2—sin% = 22 ‘ = c08.x/2 is negative and sin x/2 is positive, cos*—sin* cot 2-1 a ing numerator and denominator by sin x/2] cosS +sin% corde x —|=RHS. ‘) Ifsin c+ sin B=a.and cos a+ cos B=, prove that tan Sol. Given, sin a+ sin B= a : @ and cos a+ cos B= b @ st downloaded from jeemain. guru Trigonometric Ratios and Identities - 2.45 Now (cos a+ cos B)* + (sin a+ sin B)’= 8? + a® => cos? a+ cos*B+2 cos acosf+ sin?or+ sin?B+ 2sin axsin B= b+ a* => (cosar+ sin? a) + (cos*B+ sin? B) + 2(cos aos B+ sin asin B)= a? +b? => 2+2c0s(a-B)=a +h a+ -2 2 or eos (a B)= acosp+b tan ©, prove that cosor~ 4 2 i a+b cos (a+b) cos? Pm = b)sin? (a+b) cos? £+(a - b) org (ou? =i? 2)+0(cx 2 +n) af cos? sin? 2) (co? S -sn' 2) _ 40089 +b a+ boos downloaded from jeemain. guru 2.46 Trigonometry ERENEEEN Ircos 0 cos arcos B, prove that tan + © tan os =tan? £ @ Sol. Given, cos 0= cosa. cos, we have cos B= “* : cosa __ 0030 Now tan? B = 10088, 1 cost 2 TreosB |, 2038 cosa cost —cos0 cosa + e088 at. 2sin2** sin 20s 2+ cos 1, Prove that cot 6—tan 9= 2 cot 26. 2. Prove that £8 9=8i08 5.99 tan20, cos 0+sind 3. Prove that tan (E+0)-tn(4-0) = 2tan 26, 4. Prove that | + tan @tan 20= sec 28. 5, Prove that Lt8iN2A— 20824 _ tan 4, 1+sin2A+cos2A 1 3 6, Show that 7 : ow Ana in 10 cos lO 7, Prove that cosec A +2 cot 24 cos 4 =2 sin A. 8, Prove that “*5i024 _ cos Asin A ian(4+) cos2A cos A—sin A 4 9. Prove that cos’@sin 30+ sin? O'cos 30= 3sin40, 10, Prove that tan 0+ tan(60°+ 6) + tan(120° + @)= 3 tan 36, IL, If @and Bare the two different roots of equation d cos 0+ b sin @= c, prove that 2ab 2b? a tan(a+ B)= >—, bb cos(a+ B)= =—> in(ar+ ) a (a+ B) ae 12, Ute os SR cere oromotthwentuc arian © tetas Lem 2. cosa cos B 2 2. 2 o Concept Application Exercise 2.7 -—____-____,, downloaded from jeemain.guru ‘Trigonometric Ratios and identities 2.47 -b) 13. If tan tan = fc 4 . prove that (a~ b cos 28) (a—b cos 24) is independent of @and ¢. a+ 14. If Gis an acute angle and sing find tan @ in terms of x. tan8@ tan6 ” 15. Prove that (| + sec26)(1 +sec46)(1 + sec8 6) = sin?3A__cos?3A 16, Prove that ; sin?A cos? A = 8cos 2A. 1+ 17, If 4 = 110°, then prove that 1an A. tan 2A 18. In triangle ABC, a= 3, b= 4 and c= 5. Then find the value of sin 4 + sin 2B + sin 3C. VALUES OF TRIGONOMETRIC RATIOS OF STANDARD ANGLES 1. Value of sin15®, cos1S°, sin75°, cos75°, tan 15°, tan75°: 18 11 _ 3-1 sin] 5°= sin (45° 30°) = sin45°cos30° ~ sin30°cos45°= == -——_ = é ) v2.2 242 we Also, sin15° = cos75°=—cosi05° ial 341 Similarly, we can prove that cos] 5° = y, p se Also, cos15®=sin75°= sin 105° tan 60°=tan 45° _ Y3-1 7k 1+tan60° tan45° 341 tan15®= tan (60°—45°)= tan60°+tan45° _ V3+1 Lan TS® = tan (60° + 85° = oe ee ass cL =24V3 2. Value of sin 18°, cos18°: . . Let 8= 18°, then S0=90° > 20+30=90° 20=90°-30 sin 20=sin (909-36) sin20=cos 30 2 sin Bcos 0= 4 cos 9-3 cos @ cos? 6-3 dividing by cos 6] (J-sin? 6)—3= 1-4 sin? @ 4 sin? @+2sin @-1=0 24 f4+16 UouUsubauuy downloaded from jeemain. guru 2.48 Trigonometry sin 8= sin 18°> 0, for 18° lies in the first quadrant v5-1 : 4 sin ie., sin18 Value of cos 18°: = Sle 2v5 _ 10+2N5 2 1 g°= 1 sin? 18°= cos’ sin a ra => cos 18°= 7 N0+2v5 [+ ¢08 18°> 0] 3. Value of cos 36°, sin36°: * eos 36° =1—2 sin? 18° -2( Sai} 4 4, Value of sin 36°: , kd ag™= 1-toitaeen = | _8+2V5 _ 16-6-2V5 _10-2V5 16 16 6 sin36°= 2 Y10<2N5 : [- sin36°>0} ee ae eee ee eee cece a « sin 54°= sin (90°~36°)= conser = SH # cos 54° = cos (90° 36°) = sin 36°= <6 ho-2¥5) 4. Value of tan TE cot 7 : Let 6= ra then 20= 15° fon 'bi= EOE, 1 =cos 20=2 sin? @and sin 20= 2 sin Ocos 6] sin20 Mit = Lroosls? | TUF WI B=t 5_ a-1 sin 15° B-1 v3-1 a Wa Value of cot ak. cots te =cot (90°— 7 y= tan 12 =) Value of cot 72=: 2 Lat = 7 then 20 15° downloaded from jeemain. guru Trigonometric Ratios and Identities 2.49 14 Mb41 = 100820 _I+e0s)5° | 22 _ NPB HB B41 Now, cot ee en a (V3+V2,W2 +1) 22 Value of tan at: tan 82 = tan (90°~ 7) =cot 15 = 5+ V2 +) 2 All these values are tabulated as follows: 75° 15° 18° 22.5° 36° 67.5° 75° | f8-2V6-202 | V3-1| V5-1 | J2-v2 | Jio-25| J2+v2 | Vo+1 Ph 4 22 4 2 4 2 22 ¥8+2V6 +22 | V3+1 | fio+2vs| j2+V2 | V5+1 | ~o-2 | V-1 cos 4 2v2 4 2 | 4. 2 22 tan} (V3 -J2)(V2-1)} 2-3 fons] 5-25 V2+1 243 cot | (V3 +¥2)(V241)] 24+ V3 |fG+2V5} V2 +1 (+ 4} v2-1 | 2-3 Find the angle @ whose cosine is equal to its tangent. iven,cos@=tan@ => cos? O=sind = 1 -sin? O= sin Bor sin? 6+ sin @-1=0 = singe 5 2 Bot asin => O=sin"'(2 sin 18°) Find the value of cos 12° + cos 84° + cos 156° + cos 132°. Sol. cos 12° + cos 84° + cos 156° + cos 132° = (cos 12°+ cos 132°) + (cos 84°+ cos 156°) - 2a ate (2 = }« 2on( +156 a % = 2cos 72° cos 60° + 2cos 120° cos 36° =2sin 18° cos 60° + 2cos 120° cos 36° {82h H82)-4 downloaded from jeemain.guru 2.50 ‘Trigonometry Prove that cos 36° cos 72° cos 108° cos 144° = 4 . Sol. .cos 36° cos 72° cos 108° cos 144? = cos 36° sin 18° (—sin 18°) (cos 36°) = cos? 36° sin? 18°= (= (427 E8]4 1, Prove that sin” 48° — cos? 12° = ~ zi 2. Prove that 4 (sin 24° + cos 6°) = V3 +415. 3. Find the value of sin 47°+ sin 61° — sin 11° —sin 25°, SUM OF SINES OR COSINES OF N ANGLES IN AP. np’ —,_ sin B sin @+ sin (a+ @B)+ sin(a@+-2B)+--+sin (@+n-1B) = x sn[aver-v8] sin Proof: Let S= sin a+ sin (a+ B) + sin (a+ 2B) +--+ sin(a+n—1B) Here angle are in A.P. and common difference of angles = B Multiplying both sides by 2 sin 8 we get 28 sin ge 2sinasin + 2sinar+ P)sin 8 +--+ 2sin(ce+n=ip)sin @ Noman cn Bcf) nel 2 2 2 i ti? escssl a 3B 2sin (ar P)sin5 = cos| ar+5]—cos| += Bas ccn| et 8) 2sin e+ 26) sin > = c0s| + J-cos| as 2sin(ar-+n—1B)si B os[aren-98] os[acen-v8] Adding, we get R.HLS. of Eq. (i) os{ af) -cos|a-+can- | downloaded from jeemain. guru Trigonometric Ratios and identities 2.51 or2sin Bs ersin{arin-nf inte "2 2 2 np sin“ B = s=—2 snfavio-n 8] wal i In the above result replacing arby 7/2 + &, weget 608 + cos (a+ B) + cos (4+ 2B) +--+ C08 (a+ N=1B) = Find the valueof cos = + cos 9 +.cos 7 % 7 Sol. $= 605+ cos +c0sS™ 7 7 7 sn(5) 13 sn( 2) ( i a(t) (af) mm) aml) sin’nd Prove that sin 0+ sin 30+ sin 50+ .~-+ sin (2n~1) = ~~. Sol, sind+sin3 0+ sins 0+ + sin(2n— 1) 0= sola(Z «al (3) 04+ @n—19 ae 3 Concept Application Exercise 2.9 Sa 1. Find the value of cos % +cos=™ + cos + cos +-c0s =. W ai a i 1 2. Find the average value of sin 2°, sin 4°, sin 6°, ..., sin 180°. 3. Find the value of Sane = n ral downloaded from jeemain.guru 2.52 Trigonometry CONDITIONAL IDENTITIES Some Standard Identities in Triangle 1. tan A + tan B+ tan C= tan tan B tan C Proof: In AABC, we have A+ B+ C= => A+Ben-C => tan(4+B)=tan(n-C) => tanA+tanB=—tanC+ tan tan BtanC => tanA+tanB+tanC=tan tan BtanC 3. sin24+sin2B+sin2C=4 sin Asin BsinC Proof: (sin 24 + sin 2B) + sin 2C =2 sin (A + B) cos (4B) + sin2C =2:sin (x-C) cos (A ~B) + sin2C =2sin Ccos (A —B) +2 sin Coos C =2sin C [eos (4—B) + cosC] =2sin C [cos (4 —B) + cos {x—-(4 + B)}] =2sin C [cos (A ~B)—cos (A + B)] =2sinCx2 sin A sin B=4 sind sin BsinC downloaded from jeemain.guru 4, 0082.4 +608 2B + cos 2C=-| —4 cos cos’B cosC Proof: : (cos2 + cos 2B) + cos 2C =2.c0s (A + B) cos (A —B) +2cos?C-1 =2.cos (t~C) cos (A - B) + 2cos*C-1 2 cos C cos (A — B) + 2cos’C-1 = 1-4 c0§ A cos B cos C a . C05 A+ cos B+ cosC=1+4 sin A sin& sin€ 2 ing OD Proof: (cos A + cos B) + cos C—1 A+B A-B oo = 2 cos ——— cos +cos C-1 slept] AaB, Mel 2 2 2 = sin €(2sin sin 3) =4sin 4 sin Zain 6. sinA+sinB+sinC=4 cos4 bose cos a"? 2 Proof: (sin A+sin B)+ sinc =2sin a? cos 4#= 3 gsinc ‘Trigonometric Ratios and Identities 2.53 downloaded from jeemain. guru 2.54 _ Trigonometry Note: , tan A+ tan B+ tan C= tan A tan B tan Cis true for A + B + C= nr, wheren€ N. IFA + B+ C= 180°, prove that cos? A + cos? B+ cos? C= 1-2 cos A cos B cos C. Sol. cos? A cor B+ oe? cu HEER2A,, L4eOsDB, 0526 (cos 2A +¢082B +0082C) + ; Rie -1=4c08 Acos Beos)+2 —2cosd cosB cosC Prove that in triangle ABC, cos?A + cos B— cos? C= 1 ~2 sin A sin B cos C. Sol. cos?A + cos? B - cos* C= cos + sin? C— sin? B =cos?d + sin(C + B)sin(C- B) = 1 —sin2A + sin Asin (C-B) = 1 —sin {sind —sin(C—B)] = 1 -sin A[sin(B + C)—-sin (C- 8)] =1-2sinA sin BeosC In triangle ABC, prove that sin (B+ C—A)+sin(C+A ~ B)+sin(A + B—C)=4 sin A sin Bsin C. Sol. sin(B+ C—A)+sin(C +4 —B)+sin(4 + B—C) = sin(t—2A) + sin(n—2B) + sin (t-2C) : =sin 24 +sin2B+sin2C=4 sin A sin BsinC downloaded from jeemain. guru Trigonometric Ratios and Identities 2.55 dy | 22 de dy 2% 2 2 lax? 1-y? 1-2? ns Ix+y-+2= x92, prove that 22 z+ 1x? ey? 1-2 Letx=tan 4, y=tan B,z=tanC Nowx+y+2=ayz = tanA+tanB+tanC=tanA tan BtanC = A+B+C=nn => 24+2B+2C=2nn . => tan24+tan2B+tan2C=tan2A tan2.Btan2C 2tanA | 2tanB , 2tnC __2tanA_ _2tanB _2tanC =tan?A 1=tan? Bo 1-tan?C 1=tan? A 1tan? B1=-tan?C 2c'_ 2x dy 2 x? 1=y? 1-2? IfA+B+C=x, prove that sin? $ + sin? : sin? £ (te ‘The product of the sines of the angles of a triangle isp and the product of their cosines is g. Show that the tangents of the angles are the roots of the equation gx? = px? + (1 +g) 0. Sol. From the question, sind sinB sinC = p and cosd cosB cosC = q x- p : tan tanB tanC = 7 . 0 Also, tand + tanB+ tanC= tand tanB tanC = “ ® Now, tan tanB + tanB tanC + tanC tand _ sin A sin B cosC + sin B sinC cos A +sinC sin A cos B 0s Acos BeosC 1 . ~ 9g Usin’A + sin®B— sin?C) + (sin?B + sin? Sink) + sin? + sin?A + sin?B)] [- 4+ B+ C= mand 2'sind sinB cosC = sin24 + sin?B—sin?C] downloaded from jeemain.guru 2.56 Trigonometry i = z [sin24 +sin?B+sin?C] = age =(cos2A + cos2B + cos2C)] = J [I+ cos4 cosBeosC]= + (1 +4) 4 q q ‘The equation whose roots are tand, tanB tanC will be given by = (tand + tanB + tanC)x? + (and tanB + tanB tanC + tanC tanA)x = tand tanB tanC = 0 ore 224149, 2 0r gx = px? +(1+q)x-p=0 q q - In triangle ABC, prove that B 9 cot wanes tBu 2A a. cos? — + cos! 2 2A, 2B, aC _ Ain, BG bh cost 5+ cos! + cost 5 =242sin sin sin . IFA + B+ C= 12, show that a. sin? A+ sin? B+ sin? C= 1 -2sinA sin BsinC b cos? A + cos* B+ cos*C=2+2sin A sin B sinC a. IfA+B=C, prove that cos? A + cos? B + cos?C= 1 +2 cos A cos B cos C. hb Ifa+ B= 60°, prove that cos? a+ cos? B—cos arcos B= 3/4. |. Prove that cos’(B— 7) + cos*(y— a) + cos*(ar— ) = 1 + 2cos (B-— a) cos (y— @) cos (a~ B). fA + B+ C= nf2, show that & cotA + cot B+ cot C=cot A cot BcotC b tan 4 tan B+ tan Btan C+ tan Ctana=1 . IFA +B + C=, prove that . a tan3A +tan 3B + tan 3C = tan 34 tan 3B tan 3C A, BL AB oC h cot —+cot—+cot= = cot—cot—cot— 2 2 2 20272 cos A sin BsinC SOME IMPORTANT RESULTS AND THEIR APPLICATIONS Result 1. cos 4 cos (60 ~ A) cos (60+ A) = ; cos 34 1A + B+ C= n, prove that Proof: We have LHS :08 4 cos (60—A) cos (60+ A) 0s A (cos? 60°—sin? A) [-cos (A + B) cos (A - B) = cos? A ~ sin? B) cosa (tint 4) ~eosa(i cos? 4) - cosa( 3.0%? 4) 4 4 4 eos -34+-4c0s? A) 4(seos? A~3c0s A) 4 ws cos3A=RHS. 4 downloaded from jeemain. guru Trigonometric Ratios and Identities 2.57 Result 2, sin A sin (60 — A) sin (60 + 4) = ; sin3 Proof: We have LHS. = sin A sin (60—A) sin (60+ 4) = sin A (sin? 60°— sin? A) [- sin (4 + B) sin (A—B)=sin? 4 - =sinA (3 =sin? 4) = J sin A(3—4sin® 4) - “asin A~dsin? 4) 4 = jin =RHS. Result 3, tan tan (60° — a) tan (60° + ) = tan 307 Using the above two results, we can prove this result Prove that cos 20° cos 40° cos 60° cos 80° = = c08 20° cos(60°-20°) cos (60° + 20°) cos 60° Ex Sol. cos 20° cos 40° cos 80° cos 60° = ; c0s(3%20°) cos 60°= Joos? 60° Prove that sin 10° sin 30° sin S0° sin 70° = x. Sol. sin 10° sin 30° sin 50° sin 70° = sin 10° sin (60° —10°) sin (60° + 10°) sin 30° 1 sin? 30°= 16 Ssin(3x10") sin 30° Prove that tan 20° tan tan P= tan Sol, tan 20° tan 40° tan 80° = tan 20° tan (60°-20°) tan (60° + 20°) = tan (3 x 20°) = tan 60° sin2" A 2" sin A Proof: LHS, =cosd cos 24 cos 274 cos 2°4 «+ cos 2"*'4 Result 4. cos 4 cos 2.4 cos 274 cos 274 ++ cos 2”! A= aan [(2 sin A cos A) cos 24 cos 274 cos 234 +++ cos 2”~"A] sin : [(sin 2A cos 24.cos 224 cos 234 +++ cos 2”~"Ay" 2sin A [@sin2.A cos2.A) cos 224 cos 2° --- cos 2"-!4) sin A Fama [sin 2 (24). cos 274 cos 2° «cos 2"~'A] sin 1 2 “ = =;—— [C2 sin 274 cos 274) cos 234 --- cos 2” 'A] 2 sin A downloaded from jeemain. guru 2.58 Trigonometry [sin (2x 224) cos 234 --- cos 2”>'A} 2 sin A [(sin 2° cos 2° cos 244 «-: cos2"-14] sin A [sin 2”-"4 cos 2"~! 4] sin A [2sin2"-" 4 cos 2"-! A} A in (2%2""! 4 Paina on 0 =< sin2"A=RHS. 2" sin A 1fo= ,show that cos@cos 26 2cos 276 --- cos2”“!= 2, Oy 4 x Sol. In the above result, put 6= 2" +1 sin2"@ _* oa 2" sin @ yn » so(45) 2H cog 4% ogg 8% cog 14t _ 1 Prove that cos — cos % cos © cos = = 1, 15° a5 “8 a5 5 ~ 16 mh Sol. We have Lit.s.=cos 2% cos © cos cos (x - = 15 1s 15 15 downloaded from jeemain. guru ‘Trigonomettic Ratios and Identities 2.59 cos A cos 24 cos 27 4 cos 2° A, where A= m/I5 __[sin2* a sin 16A - [ sin 4] 2 sin A asin (t+ A) 16 sin A Iv isa=a] 1 = =RHS. 16 . 00] Prove that sin 6° sin 42° sin 66° sin 78°= Sol. sin 6° sin 42° sin 66° sin 78° = sin 6° cos 48° cos 24° cos 12° = cin co. 2 Sin 12°08 12° cos 24° cos 48° = singe ere 2 sint2° = singe S296 2 sin12° = 2sin6%cos6? _ ai 2 sin 12° Concept Application Exercise 2.11 1 a= prove that cos 2arc0s 4arcos Baxcos 14a= 2. Prove that sin 20° sin 40° sin 60° sin 80° = 2 . 3. Prove that cos 10° cos 30° cos 50° cos70° IMPORTANT INEQUALITIES Pa Sol. InAABC, tan A + tan B+ tan C= tan A tan BtanC ‘Also, 0-4 +tan 8 In SABC, tan + tan B + tan C23 V3 , where A, B, Care acute angles. [since A.M.2GM,} downloaded from jeemain. guru 2.60 Trigonometry tan 4 tan Btan C2 Ytan A tan BtanC = => tan? A tan? Btan?C2 27 => tandtanBtanC23 V3 [cubing both sides) = tand+tanB+tanC23V3 In ABC, prove that cos A + cos B+ cos CS3/2. Sol. Let cosd + cosB + cosC=x ' = ame(4 Boo 452) o1-2sin c = 2sin Soos( 422) 41—asi? x = asint S—rsin cos: Jere 2 2 This is quadratic in sin C/2 which is real. So, discriminant D=0. 4 cos? (45}-ea0-n20 = 2@-1)Se (4) = 20-1)s1 = i352 Thus, cos + cosB +cosC < 3/2 Note: Since cos A + cos B+ cos C= | + Asin Asin Fool Students are advised (o remember this as a standard result, Find the least value of sce A + sec B+ sec Cin an acute angle triangle. Sol. In an acute angle triangle, sec A, sec B and sec C are positive. NowA.M.2H.M. sec A tse B4+sec€ , 3 2 cos A +cos B+cosC But in A ABC, cos A + cos B + cos CS 3/2 = SecA fee asec 58 = sec A +secB+secC 26 downloaded from jeemain. guru Trigonometric Ratios and Identities 2.61 In AABC, prove that cosee 4 +cosee gi cosee Se 6. Sol. In ABC, we know that sin sin sin€s i 278 NowA.M.2GM. A B c cosee 5 + cosec 5 + cosec = * sloethcenlew 3 2 2 cose 4 +cosee ® 4+ cosee a ee 2 3 A B 6 cosec — +cosec— + cose = = 2 ; 2 2 2()"" A B G = cosee 5 +eosee > + cosec 26 EXERCISES = Subjective Type Solutions on page 2.85 1. Are the set of angles crand B given by a= (2» +4) nA and B=mx+(-1)" (& -4) same, where nmel A eB nS " If ABC isa triangle and tan z lan fan are in H.P., then find the minimum value of cot B/2. 3. Find the sum of the series cosec 8+ cosec 20+ cosec 40+ --- to nmterms. 4. In AABC, if sin® @= sin (A — 8) sin (B — 8) sin (C— 8), prove that cot @= cot 4 +cot B+cot C. B 3 5. In triangle ABC, prove that sin Sesing +sin £ SF. Hence, deduce that tA +B tC l) s ase. cos cos aa x ¥ z XtY a, 6. if —*— = — = 2 then show th: ~B)=0. tan(O+a) tan(0+B) tan(O+y)" sia SD er (a-B) 7. Iftan 60~ pla, find the value of Hr cosec29—4see28) in terms of p and q. 8. 1f0 2!- V2 A B e 11. If A, Band Care the angles of a triangle, show that tan? 7 +tan? > tan? z 21 downloaded from jeemain.guru 262 Trigonometry 12. Let A, B, Cbe three angles such that A = 7/4 and tan B tan C =p. Find all possible values ofp such that A,B, Care the angles of a triangle. 13. Eliminate from the equations, sin (a+x) = 2b and sin(a—x) = 2c. a 14, Iftan B= Prove that tan (or B)=(1—n) tan a 1 = nsin? sinx | sin3x | cos9x 15, Show that a conde * conde * eo dTx gM" 27*= tana]. 16. Prove thar 26°82"8+1 _ (9.09561) (200520 1)(2c0s 278 =1)-+:(2c0s2""0 1). 2cosO+1 tan2"@ - ‘ . 17. Prove that “T= = (1+-sec28)(I+-see20)(I-+see2"6)--(1+se02"8) ey ram Td Solutions on page 2.92. Each question has four choices a, b, c, and d, out of which only one answer is correct. Find the correct answer. 1. Which of the following is correct? a sin 19> sin} b sin} sin $ sin A cot $ =cos A is a. independent of A, B, C b function of A, B c. function of C d. none of these 6. The least value of 6 tan? + 54 cot? 9+ 18 is I: 54 when A.M. 2.G.M. is applicable for 6 tan? g, 54 cot? , 18. I: 54 when A.M. >GM is applicable for 6 tan? g, 54 cot” gand 18 added further. Mk 78 when tan? = cot? ¢. a. [is correct b [and Il are correct ¢. II is correct none of the above is correct downloaded from jeemain.guru ‘Trigonometric Ratios and Identities 2.63 Ssin@—3cos 7. If'Stan 6=4, then "> Is equal to ao bl M6 a6 & If2 sec 20= tan $+ cot ¢, then one of the values of 8+ gis am ba/4 «1h d none of these 9, Ifsin.x + cosec x = 2, then sin" x + cosec” x is equal to a2 b2" e2r! - aor? 10. A quadratic equation whose roots are cosec? @ and sec? @ can be a xSrt2=0 bx?-3x+6=0 e=5r45=0 none of these (Hs leeae®, then Hoes: json is equal to 2 l¥cosa VI—cosor eels — oe i. sina sina, sina sina 12, The value of cos +005 2% +008 3% 4.005% +c0s 5% +005 + cos = is 7 7 7 7 7 7 7 al h-l 0 d none of these 13. The least value of 2 sin? 6+ 3 cos* Gis 5 al’ b2 “3 as 14, The greatest value of sin* 6+ cos* @ is a 12 bl “2 a3 15. Iff(x)=cos? 6+ sec? @, then af@/@)> 1 df@)22 16. Iff(x) = sin’ x + cos® x, then range of f(x) is of] afh3] cE] am ote 4 474 4 17. Ifas3.cosx+5 sin (x= 2/6) B bA 2 4 a co) aa+is2 d none of these downloaded from jeemain.guru 2.66 51. 52. Trigonometry Iftan cris equal to the integral solution of the inequality 4x? ~ 16x+ 15 <0 and cos Bis equal to the slope of the bisector of the first quadrant, then sin (or B) sin (o-— ) is equal to 3 3 ze < ‘e be © a3 5 5 5 5 ap S08, {2 +0) 9, then the value of tan x tan y tan ztan fis equal to cos(x+y) " e0s(z—1) wet hel a2 "a2 53. Let/(n)=2 cos mV ne N, then (1) f(n* l)~f(n) is equal to 55. 56. 57. 58. 59. 60. 61. 62. 63. a. f(nt3) bf(nt2) oe f(nt I)fQ2) d f(nt+2)f(2) |. [fin triangle ABC, sin A cos B= 1/4 and 3 tan A = tan B, then the triangle is a right angled b equilateral ¢. isosceles d none of these If A and B are acute positive angles satisfying the equations 3 sin? 4 + 2 sin? B = 1 and 3sin2.A ~2sin2B=0, then A +2 Bis equal to ax n= at az 2 4 6 cot 8 5x 7 — anda p= 2 Let /(@) T+ cote and a+ B 4 , then the value f(a)/(B) is a h-4 2 a none of these 2 2 Ify= (1 +tan 4) (1 —tan B) where A - B= & then (+ 1)" is equal to * ad ba 27 d 8 If'sin(y+2—3), sin (2 +x—y), sin (++ y—z) are in A.P,, then tan x, tany, tan are in a AP. bh GP «HP d none of these Ifcos a+ cos B=0 = sin a+ sin B, then cos 20+ cos 2B is equal to a. -2sin(a+B) b -2.c0s (0+ B) ¢. 2sin (a+ B) d.2cos (a+ A) fx, x3,X3, ---)%y are in A.P, whose common difference is 6, then the value of'sin or (Sec x, sex, +e x, sec x3 + +++ eC x,y Sec x,) is sinnDae noe ; 5 See, bh Ot _ c.sin(n-1) arcosx, cosx, sin n 01 cos x, cos x, ©0S x; 608, 05.) COS, ® Iftan Z. geand ton = are in AP. and tan , y and tan & are also in A.P,, then a Qx=y bx>2 Gxey @. none of these Let x= sin 1°, then the value of the expression 1 i 1 1 tt St et tt is cos0*-cosl® " cosl°-cos 2° cos 2°-cos3° 20544°.cosas? 1S equal to ax b lik e. V2/x a xly2 Ww Let and, B be such that x< a B<3z. Ifsin a+ sin B=— 2 ahd cos o + cos B = a-B6, is » then the value of cos downloaded from jeemain. guru Trigonometric Ratios and Identities 2.67 3 2 6 a vi30 i30 65 65 6a, 1 SOG AY 244 they LX ig equal to sin@y=y) a= tan y a® be cab d none of these a b 65, Sin38+sin5O+sin 704598 5 aay * [0830+ 60850+e0870+00896 4 a tan30 b cot30 ©. tan 68 d cot60 66. Ifx,y, zare in A.P, then S2*=SRZ is equal to cosz—cosx a tany bcoly esiny d.cos y 67. Ifcos 25° + sin 25°= p, then cos 50° is a y2-p? b- 2-p? epyp-P d-py2-p* 68. ia is equal to "sin Acos. BoosB 4 a. tan (4B) b tan (4 +B) © cot (A ~B) d cot (4+ B) 69, tftan d= 128% hen tan 24 is sin B. a tan 24 = tan B b tan 24=tan? B ¢. tan 24 =tan’B +2 tan B none of these 70. Ifa+b=3~cos 40 and a~b=4 sin 26, then ab is always less than or equal to ad ht 2 a2 2 3 4 71, The value of cos? 10°— cos 10° cos50° + cos? 50° is equal to 4 1 8 = hs = a3 *3 3 “4 72, The numerical value of tan 20° tan 80° cot 50° is equal to 1 ‘ 1 a 3 bh « W3 a B a3 73. Iftan? @= 2 tan? 9+ |, then cos 20 + sin? p equals a-l ho el d none of these 74, The value of cot 70° +4 cos 70° is as ni . W3 , + 75. If.x, and x; are two distinct roots of the equation a cos.x +b sin x= c, then tan a. is equal to as at « at b @ c alo downloaded 2.68 76. 77. 78. 79. 80. 82. 85. 87. 3r . AE. += b we (« =) from jeemain.guru ©. Trigonometry Given that (1+ V1+x)tan y=1+Vi—x. Then sin 4y is equal to a dx ba Gx d none of these If cos.x = tan y, cos y= tan z, cos 2= tan x, then the value of sin x is a 2cos 18° b cos 18° ¢. sinl8° d 2sin18° If sin 20 = cos 3 @and Gis an acute angle, then sin @equals, ¥5-1 -» {5-1 V5+1 ~V5-1 a b-|e— c a 4 4 4 4 5 ; % san 02 If 8, and @, are two values lying in [0, 27] for which tan @= A, then tan zm pis equal to ad h-l a2 di If'tan 6= Yn where n ¢ N, 22, then sec 26 is always ‘a. arational number bh anirrational number ¢. a positive integer a negative integer . 7 & I sin.x+ cosx= - where € 4, then tan 2 is equal to vi Ni-2 a na 3 3 9H 9 St. ‘The value of sin? gtsin’ - + sin’ none of these 1 al bh2 » i= da 2— 2 3 ) + Vasin® x-+sin® 2x is always equal to « al d none of these cos'x sin2x= 214-Sin(r2) vive R, then = a n=5,a,=12 bn=5,a,= 1/4 @n=5,a,= 1/4 The value of cos 2(0+ ) +4 cos(O+ @) sin Osin +2 sin? @ is 1. independent of @only . 'b independent of only ¢, independent of both 8 and ¢ dependent on O and ¢ tfcos 2 B= 2284+ then tan A, tan B, tan C are in cos(A—C) a AP. hap HP. none of these Itcosx = 2O08Y=! wherex,y@ (0,2); then tan cot2is equal to 2=cosy 22 1 1 a v2 b v3 « ad 2 3 Iftan x= bia, then \(a+b)ha—b) + (a=) Kab) is equal to a 2sinx/sin 2x h 2cosx/Vcos2x — . 2 cos x/-Vsin2x d.2sinx/Yeos2x downloaded from jeemain. guru = Trigonometric Ratios and identities 2.69 89. If isa root of 25 cos? @+5 cos @~12=0, e t 1 3 WS. 5 aks downloaded from jeemain. guru ‘Trigonometric Ratios and identities 2.71 114, fa cos? @ +b? sin? 6+ Ja? sin? 6 + 5? cos? 8, then the difference between the maximum and minimum values of 1? is given by a a+) bh 2a? +0? «(a+b d(a~by HS. If (sin + cos.x)? + Asin x cos x= 1 holds Vx R, then the value of & equals a2 b2 e2 a3 116, The range of & for which the inequality k cos*x— kcosx+120 Vx € (ee), is aks b a sks4 ak>4 a $skss 117. The minimum vertical distance between the graphs ofy*2 + sinx and y= cos.xis a2 bt « v2 a2-v3 118, 1f = 3orand sin £ +b ‘The value of the expression a cosec orb sec ris a b 2ya? +b? cath d none of these Yo? +6? 119. Ifthe equation cot*x—2 cosec? x + a= 0 has at least one solution, then the sum of all possible integral values of ‘a’ is equal to ad h3 «2 ao 120. If the inequality sin?x +'a cos x + a? > | + cos x holds for any x € R then the largest negative integral value of ‘a’ is at 3 2 7 del 121. Intriangle ABC ifangle Cis 90° and area of triangle is 30 sq. units, then the minimum possible value of the hypotenuse ¢ is equal to a 30/2 h 60V2 «. 120V2 a 30 122. ‘The distance between the two parallel lines is 1 unit. A point ‘A’ is chosen to lic between the lines at a distance ‘¢ from onc of them. Triangle ABC is equilateral with B on one line and Con the other paralle! line. The length of the side of the equilateral triangle is 2 a Ae sav b 2, eaaet «. 2d? a +1 a Ja? -a+1 123. Given that a, b, ¢ are the sides of a triangle ABC which is right angled at C, then the minimum value of 2 £42) is ab a0 ha “6 a8 124. Lety=(sin x+ cosec x)? + (cosx + sec x)'+ (cos.x + sec x)’, then the minimum value of y, Wx R, is a7 h3 «9 ao Multiple Correct Answers: Type Solutions on page 2.116 Each question has four choices a, b, c, and d, out of which one or more answers are correct. 1. Ifcos Bis the geometric mean between sin aand cos a, where 0 < @, B< 72, then cos 2B is equal to 2s? | 2 tees |Z ie |S. Az a-zsi?(E-a) a-zae(Zea) ezie(E+0) azew'(-a) 2. 1f0< O tan Be ((ifdefined) fh tan @¢ Q=> sin 20, cos 2@and tan 20 Q (if defined) «.ifsin @€ Qand cos 8 Q=>tan30c QNif defined) d ifsin 8 Q=»cos30€ Q 5. Which of the following quantities are rational? asin (UE) sin (2% ty cose (2 seo (42 12 12 10 3 esis (4) -0 (2) a (1+co12)(isn #)(1+ e018). 6. Inwhich of the following sets the inequality sin®x + cos® x > 5/8 holds good? a. (A, 18) h (78, 5/8) ©. (w/4, 31/4) (7708, 9778) 7. Which of the following inequalities hold true in any triangle ABC? B asin 4 sn 8 sin So w cor A cos & cor SMA este A s3ut 2 oat Sc? a cot A seed B reat 252 8. For a= 2/7 which of the following hold(s) good? @ tan @tan 2a tan 3@= tan 30— tan 2@-tan o cosec a= cosec 2ar+ cosec 4or €. cos cos 2or+ cos 3a= 1/2 8 00s crcos 200s 4a= 1 9. Which of the following is/are correct? a (tan. x)" > (cot x)", Vx (0, 1/4) bb qimeovers < smote, Y xe (0, 1/2) 6. (1/2IM6°82)-< (1/3)! Ye (0, 12) a 2a) > hina) -Y xe (0, 2/2) 10. Which of the following do/does not reduce to unity? (180° + A) cot (90° + A) cos (360° — A) cosec A * tan 180° + A) tan (00° + A) A) pn 8A) tan 90" + A), 008. sin (180° + A) ‘cot A sin 0° + A) sin 24°cos 6° — sin 21° cos 39' 6° cos 24° 0s 51° sin 69° cos (90° + A) sec (— A) tan (180° - A) sec (360° + A) sin (180° + A) cot (90° = A) downloaded from jeemain.guru Trigonometric Ratios and Identities 2.73 11. Which of the following identities, wherever defined, hold(s) good? @ cot a tan a= 2 cot 20 b tan (45° + a) —tan (45° a) =2 cosec 201 ¢, tan (45° + af) + tan (45°— @) = 2 sec 2a d tan @+cot @=2 tan 2a 12. A circle centred at O has radius | and contains the point 4. Segment AB is tangent to the circle at A and ZAOB = @.\f point C lies on OA and BC bisects the angle ABO, then OC equals Fig. 2.36 cos? @ 1 1+ sin@ © Tr sin@ sin 8 2 cos” a. see O(sec @-tan @) b a 13. The expression (tan’.x +2 tan?x + 1) cos? x when x = 1/12 can be equal to a 42-3) h (V2 +1) ©. 16 cos? w/12 4 16 sin? w/12 14, Let a, Band y be some angles in the first quadrant satisfying tan (a+ B)= 15/8 and cosec y = 17/8, then which of the following hold(s) good? aatrpryan bh cot acot Bot y= cot a+ cot B+ cot y ¢, tan or+ tan B+tany=tan cetan Btany —@_ tan artan B+ tan Btan y+ tan y tan a= 1 18, (a+2)sin 0+ (2a-1)cos @=(2a+ I) iftan cis a 3/4 has ©. 2a? +1) 4 2al(?-1) 16. Let f(x) = log (logs (log; (sin x + a))) be defined for every real value of x, then the possible value of ais a3 b4 05 a6 17. Ifb> 1, sin¢>0, cos ¢> 0 and log,(sin ¢) = x, then log,(cos 1) is equal to a Floestt-2*) b 2 log (1-5) ¢. log, VI-b** log,(1—b"*) a yi-x? 3 8 is satisfied by a. x= cos (2 bx = cos/ 2 ex = cos 23% x= cos{ 12% " 18 18 18 18 downloaded from jeemain. guru 2.74 19. Trigonometry If'sin (x-+ 20°) = 2 sinx cos 40° where x ¢ (0, 7/2) then which of the following hold(s) good? a cos 2x= 1/2 hh cosec 4x=2 e seo = V6.- V2 atan> = 2-J3) Reasoning Type Solutions on page 2.122 n has four choices a, b, ¢, and d, out of which only one is correct. Each question contains STATEMENT | and STATEMENT2, 10. nt. |. Statement 1: Ifx+y +2 =x ‘a Both the statements are TRUE and STATEMENT 2 is the correct explanation of STATEMENT 1 'b Both the statements are TRUE but STATEMENT 2 is NOT the correct explanation of STATEMENT 1 c. STATEMENT | is TRUE and STATEMENT? is FALSE dl STATEMENT | is FALSE and STATEMENT? is TRUE , then at most one of the numbers can be negative. Statement 2: Ina triangle ABC, tan 4 +tan B+ tan C= tan A tan B tan C and there can be at most one obtuse angle in a triangle. ‘Statement 1: cos} 1. nB+tanC tan Bian C=1" Statement J: tan 5° is an irrational number. Statement 2: tan 15° is an irrational number. Statement 1; sin 7/18 is a root of 8x ~ 6x + 1 =0. Statement 2: For any Be R, sin3@=3 sin 6-4 sin’ 8. Let / be any one of the six trigonometric functions, Let 4, Be R satisfying /(24)=/(2B). Statement 1:4 = n+ B, for some ne Z. Statement 2: 27ris one of the period of f: Statement 2: In any triangle, tan A= downloaded from jeemain. guru 13, 14, 15, 16. 17, 18. 19, 20. tat) ‘Trigonometric Ratios and Identities 2.75 . Statement 1: sin3 4 sin Ocos O(1 —2 sin? 6) = sin 0 (4sin’@— 3) Rejecting the value sin @=0, we get 4cos @(1 ~2 sin? 6)=4 sin @—3 downloaded from jeemain. guru Trigonometric Ratios and Identities 2.7 = 16 cos? {1—2 sin? 6) = (4 sin? @-3)? = 16 (1 sin? (1-4 sin? 8+ 4 sin* @) = 16 sin 6-24 sin? +9 => 64 sin® 6-112 sin* 6-56 sin? @-7=0 This is cubic in sin? @ with the roots sin*(21/7), sin*(47/7) and sin*(87/7). ‘ Qn ax. The sum of these roots is sin? sin? + sin Now answer the following questions 10. The value of (un? + tan? 2 +tan? =) (ox? $+ 000? 2% + cot? 2) is 7 a 105 3s 210 4 none of these tan? © + tan? 2% 4 tan? 3% 11. The value of ——— is cot? = + cot? + cot? 7 7 7 al h 398 © 25 none of these 12. The value of tan? tan? 2 tan? 2 ig . TF z a-3 b-7 a5 d none of these For Problems 13-15 Analtitude BD and a bisector BE are drawn in the triangle ABC from the vertex B. It is known that the length of side AC= |, and the magnitudes of the angles BEC, ABD, ABE, BAC form an arithmetic progression. 13, Thearea of circle circumscribing AABC is ae nz a an 8 4 2 14, Let‘O" be the circumcentre of AABC, the radius of the circle inscribed in A BOC is 1 1 1 1 a 1— /_ => as WB a3 23 2 15. Let B’ be the image of point B with respect to side AC of ABC, then the length BB" is equal to 8 v2 1 3 NS Pees : we ay : ay ay Cee MLEcUms Solutions on page 2.129 Each question contains statements given in two columns which have to be matched. Statements (a, by, d) in column I have to be matched with statements (p, q, 6,5) in column I. If the correct matches are a-p, a-s, b-q, b-r,¢-p,c-q and d-s, then the correctly bubbled 4 * 4 matrix should be as follows: Pars 1]@@O®@ +/©@O@ d ®OO® ©O©O® downloaded from jeemain. guru 278 ‘Trigonometry 1. If'cos @—sin @ = : where o<0< = Column tT Column il a. (cos O+ sin 6)/2 p 3 a bh sin20 2. Forall real values of @ Columal Column It a A=sin® 0+ cos! @ p 4e[-1,1] sSentendil 3 b A=3.cos? 6+ sin’ @ 4 Ae] Fil c. A= sin? @—cos 0 r. Ae [2V2, 0) @ A= tan’ 8+2col?@ s. Ae [1,3] 3. Ifcos a+ cos B= 1/2 and sin a+ sin B= 1/3 Column It vB 2, WS, ak ez bas H 5 B downloaded from jeemain. guru Trigonometric Ratios and Identities 2.79 Column} Column I a. sin (410°—A) cos(400° + A) + cos (410° A) sin(400° + 4) has the value equal to cos? 2° aT i equal to €. sin (— 870°) + cosec (~ 660°) + tan (~855°) +2 cot (840°) + cos (480°) +see (900°) 4 Ifcos 6= ‘ where 6€ ( ; 2) and cos $= 3 where pe (0. x then cos (0 ¢) has the value equal to p-! Column ‘Column II ‘& The maximum value of (cos (24+ @) + cos (2B+ 0)}, where 4, B are constants, is p 2sin(4+B) 'b The maximum value of {cos 2.4 + cos2.B}, where (A+B) is constant and 4, 8 € (0, 7/2), is ¢. The minimum value of {sec2.A+sec 2B}, where (A + B) is constant and 4, Be (0, 7/4), is q 2sec(A+B) 20s (A+B) d. The minimum value of {tan + cov@—2cos2(A+B)) , where 4, B are constants and @¢ (0. 1/2), is Column s. 2.cos (A ~B) Column Il a. cos 20°+ cos 80°— V3 cos 50° 3a oa cne® an 4n Sa 6a bcos 0° + cos + c0s— + cos + cos + cos + cos &. cos 20° + cos 40° + cos 60°—4 cos 10° cos 20° cos 30° cos 20° cos 100° + cos 100° cos 140° cos 140° cos 200° downloaded from jeemain. guru 2.80 4 Trigonometry 2 Column Column it ‘@ Suppose ABC is a triangle with three acute angles A, Band. —_p 1* quadrant ‘The point whose coordinates are (cos B~ sin A, sin B —cos A) can be in the b 1f25"8> | and 3° <1, then 0€ . g 2" quadrant ¢. [cos x + sin x|=|sin x| +|c0s x| 1. 3° quadrant inA_ 1 sae a <** =, forall permissible values s. 4" quadrant cosA cosa of A, then 4 can belong to 8 Column Column I a fx? +7 = 1 and P= (3x—4x°)* + Gy—4y), then P is pl equal to B Ifa+6=3—cos 40and a—b=4 sin 20, then the maximum a4 value of (ab) is ‘. The least positive integral value of x for which nS 3.cos 0= x*—8x + 19 holds good is a = a ifx= ay and = 2L2F where Aisa real parameter, 8.8 then x? ~xy+y? lies between (a, b] then (a + 6) is 9. Column Column tt a, In ttiangle ABC, 3 sin A+4 cos B= 6 and po 3.cosA +4 sin B= |, then ZC can be b Inany triangle, if(sin.A + sin B+ sin C) 30° (sin A + sin B— sin C) = 3 sin A sin B, then the angle C ¢, If8 sin x cos*x ~ 8 sin*x cos x= 1, thenx= 165° 4. ‘0’ is the centre of the inscribed circle in a 30° — 60° 90° s. 15° triangle ABC with right angled at C. Ifthe circle is tangent to AB at D, then the angle ZCOD is Integer Type ff ir ptey= 12 8in 20 + 60828 Teas 2g Wen valueof 8/(11°)-/(34*)is Solutions on page 2.135 - LEFG)=2(7 cos.x+ 24 sin x)(7 sin.x-24 cosx), for everyx € R, then maximum value of (f(x))"" is. . Inatriangle ABC, ZC= = Af tan (3) and tan (3) are the roots of the equation ax? + bx-+e= 0 (a#0), +b then the value of “= (where, a, 6 care sides of A opposite to anges 4, B,C resp.) is : downloaded from jeemain.guru 8. 10. nM. 12, 13, 14, 15, 16. 7. 18, 19. 20. a. 22. 23, ‘Trigonometric Ratios and identities 2.81 If (1 + tan 5°X(1 + tan 10°)(1 + tan 15°)...(1 + tan 45°) = 2", then the value of ‘&' is (140°) sec(280*) | sec(340") The value of V3] —See2209, cosee(20") cot(200°) — tan(280%) cot(200°) Ifx, ye R satisfy (x + 5)? + (y— 12)? =(14)", then the minimum value of is ‘Suppose x and y are real numbers such that tan x + tan y= 42 and cot x + coty=49. Then the prime number by which the value of tan(x +y) is not divisible by 5 is Let 0< a, 6, ¢, dS where 6 and c are not complementary, such that 20s. a +6 cos b+ 7 cos c+ 9 cos d= 0 d@ and 2 sin a~6 sin b+ 7 sin ¢=9 sin d= 0, then the value of 35224 jg cos(b +c) Suppose 4 and B are two angles such that A, 8 € (0, 7), and satisfy sin A + sin +05 B= 0. Then the value of 12 cos 24 +4 cos 2B is crand Bare the positive acute angles and satisfying equations 5 sin 2B=3 sin 2aand tan B=3 tan a simultaneously. Then the value of tan + tan Bis ; ‘The absolute value of the expression tan + tan 2% + tan 2% + tan 13% is 16 16 7 16 16 1 1 ——+—1_is cos 290° J3sin 250° Vand cos 4 The greatest integer less than or equal to 1 re ff sin® x+ cos®x The maximum value of y= The maximum value of cos? (45° + x) + (sin x - cos x)* is sin’ + co: F The value of 9——~———-— is sin®t + cos®t—1 The value of cosecl0° + cosec50° - cosec70° is The minimum value of J@sin x — 4cos x - 10)Gsin x + 40s x—10) is ‘Number of triangles ABC if tan A = x, tan B= x +1 and tan C= 1 ~x is I -1 If logygsin x + logygcos x =~ 1 and logyg(sin x + cos x) = fet , then the value of ‘1/3? is sin 1° + sin3° + sin5® + sin7° The value of 1e value o} aa cos 1°-cos 2° Ina triangle ABC, if A - B= 120° and sin sin Bein 4 then, the value of 8cos C is. 1 Inatriangle ABC if tan A= i, tan B= k+ > andtan C= 2k+ re then the possible value of [4], where [+] represents greatest integer function is If sin'x cos 3x + cos*x sin 3x = 3/8, then the value of 8sin 4x is downloaded from jeemain. guru 2.82 Trigonometry - Solutions on page 2.141 Subjective 1. Iftan = and tan B= ——, find the possible values of (a+). (UTE, 1978) m+) 2m+t 1 nf y= ina = 2 tox=% 1975 2. a. Draw the graph ofy = Fe (Sins +e083)froma= —F tox= 5 (ITSEE, 1979) b. Ifcos (a+ B)= $sina-P= and @,, Blie between 0 and 7/4, find tan 2a. 3. Prove that 5 cos 0+ 3 cos (0-2 +3 lies between —4 and 10. 4, Given a+ B~ y =z, prove that sin? a+ sin? B— sin? y= 2 sin asin Boos y. (ITAJEE, 1980) 5. For all @in (0, 7/2] show that cos (sin 8) 2 sin (cos 8). (NTJEE, 1981) 6. Without using tables, prove that (sin 12°) (sin 48°) (sin 54°) = 1/8, (IT-JEE, 1980) 1. Show that 16c0s{ = Jcos (= cos (2 Jos (f (UT-JEE, 1983) 15 15) as 15 8. Prove that tan +2 tan 2044 tan 4er+ 8 cot 8ar= cot a. (IT-JEE, 1988) 9. ABC is a triangle such that sin (24 + B) = sin (C- A) =~ sin (B+ 2C)= 1/2. If A, Band Care in A.P. determine the values of 4, B, and C. (IT-JEE, 1990) 10. Show that the value of = x. wherever defined, never lies between : and 3. (IT-JEE, 1992) an 3x e 2k on 41. Prove that Yn A)eos—* =—F, where 23 isan integer. (UTIEE, 1997) kel a 2 2 . 12, Find the range of vatues of ¢ for which 2 sin p= I=22452 (ITE, 2005) 3 13. Find the maximum value of the expression —————!____ (NT-JEE, 2010) sin? @-+3sin @ cos +5.cos*@ Objective Fill in the blanks 1. Suppose sin’? x sin 3x= ' C,, cosmx is an identity in x, where Co, Cy, ...,C,are constants, and C, #0, then the value of n is__. (UTE, 1981) The side of a triangle inscribed in a given circle subtends angles a, Band yat the centre, The inimum, value of the arithmetic mean of cos (« +), oos( | and oos(y +2) is equal to__. (UT-JEE, 1987) downloaded from jeemain. guru Trigonometric Ratios and Identities 2.83 3m, Sa Im On, Wat, 13H 3. The value of sin“ sin 7-sin-7 sin sinc tsin == sin is equal to (UT-JEE, 1991) 4, If K= sin (x/18) sin (52/18) sin (77/18), then the numerical valueofKis___..__(IIT-JEE, 1993) 5. IfA>0,B>@and.4+ B= 23, then the maximum value of tan A tan B is__. (UT-SEE, 1993) 6. Ifcos (xy), c08.x and cos (x+y) are in H.P, then cos.x see (3) — (UTEE, 1997) True or false 1. Iftana= 2 then tan 24 = tan B. (IT-SEE, 1983) si Multiple choice questions with one correct answer 1. If tan @+ sin @= m and tan @~sin @= n, then a mt =4mn b m+ =dmn mar =m +P do m1? = 4inn (IT-JEE, 1970) 2. Iftan O= -4. then sind is (IT-JEE, 1979) a-founet —w-tort Stator 4 none ofitese 3. a+ B+y=22, then atone tan Stan = ton ton 2 ent tan 2 tan 8+ tan 4 n+ tan Y tan 21 oe ee 22 2 ee % tang tan be tan ie =-tan Suan Bind none of these (UTAJEE, 1979) 4. Given A= sin? + cos* 6, then forall real 6, a 1SAS2 b3/4SAS1 ©. 13/6541 a 3454S 13/6 (NT-IJEE, 1980) I 5, The value of ( + cos ic + cos =) (i + cos #) ( + cos %) is al baa 18 43/8 (TIE, 1984) 6. The value of the expression J3 cosec 20°—sec 20° is equal to : a2 b2sin20%sin40® 4 @ 4 sin 20%sin 40° (INT-JEE, 1988) 7. 3 (sin x—cos x)' +6 (sin x + cos x)? + 4 (sin® x + cos® x) is equal to . all br «13 414 (TYEE, 1995) 8. sec? @= te is true if and only if (ty) axtyz0 hr=y,x20 ery dx#0,y40 (IT-JEE, 1996) 9. Let/(6)= sin O(sin 6+ sin 36). Then /(6) is a 20onlywhen 620 bh>h>h bar B>hrh Gh>hr>h>ty dar ty>h>h (UTJEE, 2006) Multiple choice questions with one or more than one correct answers 1. The expression afin (3« “ a} sin’ +o]- 2 [sns(La+a)+sin*or a) i equal to a0 bi «3 d none of these (IIT-IEE, 1984) 2. For0. cot—cot—cot= = cot — +cot = +c0t = 20202 2 2 2 But tan, tan, tan are in H.P. @ A B.C cot —, cot =, cot = are in A.P. ey $0,cot4+cot = 20012, 2” 2 2 v.65 Dbermet catoa ton’ asco” arcu tea a3 => GM of cor andoot £ = cot A c t= +cor= cog to and AM. of cot and cot & = 2 2 2 downloaded from jeemain.guru 2.86 4,, B.C . Let sin<+sin-—+sin— 2 2 Trigonometry ButA.M:2GM. => cot 22 v5 Therefore, the minimum value of cot B/2 is V3. cosec 0= sind 1 sino/2 2 nal -snain sinasin(2) - sindsin’ ‘ =. cosec 0= cot Scot Similarly, cosec 20= cot @—cot 20 cosec 40= cot 26—cor4e coseo 2""! 9= cot 2"-? 8- cot 2"-1 8 Therefore, sum = cot =cot2""" 8 Let cot @= cot.4 + cot B+cotC => cot @— cot A = cot B+ cat C sin(A~6) _ sin(B+C) - sin Asin@ sin BsinC Similarly, sin (B— 6) Set Bint wo and sin (Ca) = Sm Csind * . il) By multiplying corresponding sides of Eqs. (i), sii’ @= sin (A ~ 6) sin (B-6) sin (C~ 0) i), and (iii), we have 2 + 9g AB 405 A*F = 4 or 2sin 4 A-B. A+B sin’ => 2sin? tke AtB a cos et since sin 4*8 isreal, 4cos* A= -ga—ty20 . A-B = Ak=1)Scos? Sl =>ks32 downloaded from jeemain. guru ‘Trigonometric Ratios and Identities 2.87 Hence; 2sin: AEB pRB. sin ARE at 4 4 4 A+B ArB “4 A+B mHC+A-B n4C-A4B 1 = Asin ATF gin FECT ANB i PACHA ch 4 8 8 2 + n-C. n-B. R+C n4+B tA 1 sin = cos=—= cos = cos" < — 4 4 48 => 2sin = 4sin 4 x _ tan(@ +a) ii Here ===" , 6 Here = (ane) BY Somponendo end dividendo, we get x+y _ tan(O+a) +tan(O+f) _ sin(20+a+B) x-y tan@+a)—tan@+p) sina) 2. 2sinX(e- 8) amy = sin(20 +a: + B) sin(a— B) [cos 2(6 + B)—cos 2(0 +a2)] @ Simitarly, asin B- n= Ffoos2(0 +7)—0s2(6 + B)] @ and = sint(y—an = le0s2(0-+a)~cos 200+ 7) ii) Adding Eqs. (i), (ii), and (iii), we get L.H.S. 7. Here, we have tan 60= plq sin6@_p a _Vr+e [Page meee = =k cos6O g i ge ee Nowy= Hi peosec0-gsee20)~ Hf p___4 ) sin 20” cos 26, _ 1) peos28~qsin20 sin 20.c0s 20 = 2 2ksin 6000s 20~2k cos 60sin2 | _ , sin(60-28 ‘4sin 20 cos20 sin 40 8. sin a+ cos a+ (tan a+ cot @) + (sec a+ cosec &)=7 1 sina-+cosee sincrcosa: sin cose = (sinartcos.a)+ 1 4 28 4 —— |= 49-——— + =taa) 2a sin? 20 = (inatcosay(i+ > (sina sin 2a downloaded from jeemain.guru 2.88 Trigonometry Let sin 20=x = (l+x)(2? $4244) =49x7 28044 => den(i+4+4)- 49-. => x3-44-36x=0 => 2x ~44x—36=0. (asx=sin2a#0) 9. We have ot 2 a 1 +cot.0— cof 14 cot 5 a eos = 1+ eot a their G.M., we have is, ei Visin(x+ © I ET = TT (+4). par Fr -1 L pd = ee og A 1. wenave 443 + 5-4, so shat downloaded from jeemain.guru ‘Trigonometric Ratios and Identities 2.89 now nt st 2 ua? © = £[S2une4 ~ Y2tan + tan 3) walle f= me BY (oe $00 SF -( anon ao 12, A+B+C=n = B+C= 7 20<0,C <2 Alsotan Bian C=p sin BsinC _ p > cosBeosC 1 cos B cosC -sin BsinC _1- = CosB cos +sinBsinC 1+p cos(B +C) “1 ~, py BEB SG) es cos(B-C) I+p L+p . = AP ncos (B-C ( V2 (p -1) =<) E a Since B or C’can vary from 0 to 37/4, we get 0sB-C< hae ! . ) - (p-1) Also #192 (p=!) <4 v2 (p-1) (p-(2u)) = “———/29 (p-1) = p (V2 +1) 13. Adding sin(a + x) + sin(a—x) = 2(b +e) => 2sinacosx=2(b +c) bre = cosx= —— i sina 7 © downloaded from jeemain. guru 2.90 Trigonometry Subtracting, we get sin(a+x)-sin(a—x)=2(b~c) = 2cos asinx=2(b-c) b-e cosa ‘Squaring and adding Eq. (i) and Eq. (ii), we get er (+e), be) sin’a — cos*a = sinx= @) nsincecosar 14, tan B= ———_ I=nsin? o nsin acoso sy =o a {dividing numerator and denominator by cos’) cos? or _ titan or ntana __ntan oe 0 sec?a=ntan?a I+tan?@—ntan? a 1+(1=n)tan? a tana ~t Now, L.H.S.= tan(ar— y= an = tan B T+tanor tan B nian a ET + an? A ome [From Eq. (i)} 1+tane— 2S 14(1=n) tan? 3 tan or + (I=n) tan? a — ntana (1=n) tan +(1 = n)tan? te 2 —" So = (=n) tan or 1+ (1 —n)tan? @ + ntan? @ 1+ tan’ a 15. L.H.S. contains x, 3x, 9x and 27x, whereas R.H.S contains 27x and x only. So, we will manipulate terms as shown below RHS= 4 [tan 27x —tan x] : (tan27x-—tan9x) + (tandx—tan3x) + (tan3x—tan2)) = Affsin27x _sindx)., (sindx _ sin3x) | (sin3x _ sinx 2[\cos27x cos9x}" (cos9x cos3x) \cos3x cosx 2 (27x -92) , sin (Ox=3x) | sin eo) 2| cos 27xcos 9x “cos9xcos3x — cos3xcos.x. of sint8x sin6x ,_sin2x 2Lcos27xcos9x ” cos9xcos3x ” cos3x cosy. downloaded from jeemain.guru . We have to prove . We have to prove that ‘Trigonometric Ratios and identities: 2.91 — 1} 2sin9xcos9x | 2sin3xcos3x , 2sinxcosx 2|cos27xcos9x cos9xcos3x cos3xcos.x = Sindx , sin3x | sinx _ sinx | sin3x | sinSx cos27x cos9x cos3x cos3x cos9x cos27x =LHS. 2cos2" @+1 2cosd+1 or2cos 2” 8+} =[(2 cos 8+ 1) (2 cos 8— 1)] (2 cos 20— 1) (2 cos 2? 8-1)... (2. cos 2"-! 6 1) Now [(2.cos 0+ 1) (2 cos @—1)] (2. cos 20~ 1) (2 cos 2? 8-1) ...(2c082"-' 8-1) = (4-cos* @~ 1) (2 cos 20~ 1) (2.082? @- | ... (2.¢052"~! 8-1) = (2. cos 20+ 1) {2 cos 20-1) (2 cos 2? @~ 1)... (2cos2”"' 8-1) {using cos 28=2 cos? @—1] = (40s? 20-1) (2 cos 2? 6-1)... (2 c082”"! @-1) . = (2 cos 2? 8+ 1) (2 cos 2? 8-1)... (2. cos2"-! 8-1) = (4cos? 2? 8-1) (2 cos 23 B= 1)... (2 cos 2”! 8-1) = (2 cos @— 1) (2. cos 26-1) (2 cos2? @~ 1)... (2cos 2"~! A= 1) = (2.cos 2"-! 8+ 1) (2 cos 2”-' 6-1) =4 cos? 2"-! 6-1 =2 cos 2” 0+! mae = (1 +se026)(1 + sec 2? 6) (1 + see 2° 6)... (1 + sec 2” 6) an or tan 2" 6 = tan O(! + sec 2) (1 + sec 2? 6) (1 + sec 2 6)... (1 +sec 2” 6) Now tan 6(1 + sec 26)(1 +sec 2? 6) (1 + sec 2° 6)... (1 + sec 2” 6) . nano (e022) (1+ sec 2? 6) (1 + sec 2? 6)... (1 + sec 2" 8) 00328 cos0| “cos 20 = (tan 26) (1 + sec 2? 6) (1 + sec2? 8)... (1 +sec 2” 8) ; 2 “(es 2) tae 2! +s! On tan 2 =(tan26) (2224) (1+ see.2? 8) ...(1 + sec2” 6) cos 2 ~om2o (2222) (1+sec23 @... (1 +sec2” 6) cos = (tan 2? 6) (1 + sec 27 @)... (I +sec 2” 6) =tan2""' (1 +sec.2” 8) tan 20-1 ( 1te082"0 cos2"6 = tan 2"! onze =tan2"0 downloaded from jeemain. guru 2.92 ere cne 1.b. Since /(x)=sin x is an incres 3.d. 4b. ayy We have =? 142) A B ( ce A.B . 2sin & cosec — | sin——cos—cos 2 in) Trigonometry 1g function for 0 21 (7 A.M.2GM,) ay 2ky 2) 2: Q ety xy Pay? 2ay Since 0 0 But sin? @< 1, Therefore, ~ sl =xsy. We have 1 +sinx+sin?x+---co=4+ 273 cosxtanx 1 sin x. eae + ¥ tanx 1 +cosx = Sin x , cosx (1 te0s x) +sin?x _ cos x(l+c08 x) +(1=cos?s) _ a, tia. & sin x (I+cos.x) sin x(I+ cos x) ko sinx kak 2 = 2sin A cosee 2 (cos A+B 6054 cos 9) cos A 205 2 2 A = 2sin—cosec 2 downloaded from jeemain.guru Trigonometric Ratios and Identities 2.93 6.b. Applying A.M. 2G.M. in 6 tan? g, 54 cot? ¢, 18, we get 2 2 Stan" o+S4c0t G+18 (6 54x 18)218 This is true if 6 tan? = 54 cot? p= 18 = tan? ¢=3 and cot? g= 1/3 Therefore, Land Il are correct. 7.0. Stan @=4 => tan a= 4 58nd Ssin-3c0s@ _ cos a) SsinO+2cos SinO, Stan+2 4,5 6 Bb, 2sec2O=tang+cotg SO 2 in? b+cos* > = 20 singcosp 2 1 » ~~. 1 _ ©0826 sin cos => cos20=sin2¢ => 20=90°-29 = org=4 9.a. sinx+cosecx=2 => (sinx-1P=0 => sinx=t => sin’x+cosec’x=1+1=2 1 10... sec? 6+ cosec? @= ——+——— = —=—2> . sin?@ sin? 26 cos? @ Also, sec? @ cosec? @= 2 sin? 26 Hence, the only equation which can have roots cosec? @and sec? @ is x ~ 5x +5=0. tip, [208 , [feos _1-cosa+1 +08 “Viscose Vi=cosa i eata a 2 jsinx] —sina (since m < or<31/2) downloaded from jeemain. guru 2.94 12.b. 13. 14.b. 15.. 16a. 18.4. 19.d. 20.b. = Trigonometry > 2n 3a 4a sa on In We have cos 5 +03 toss +008s—+cos=— +cos—+cos— 7 7 7 7 nL Or Qn St an. 4m = | cos—+cos— |+| cos—+cos— ] +] cos=--+cos— |+cos x 7 7 7 7 7 7 x a Qn) an ( 3x 30 =) =| cos=~cos= |+| cos—--cos— | +] cos—-—cos— |+cosm 7 7 7 7 7 v1 =cos z=-I 2 sin? 8+ 3 cos? 8=2(sin? preadoseition 2+cos* 022 [- cos? > 0] sin’ @+ cost @= (sin? @+ cos? @)*—2 sin? @cos* @= | -2 sin? @cos* AS 1 ‘We have . S() = cos? 6+ sec? = (cos O-"sec 6)? +2 cos O sec O= 2 + (cos 6— sec O22 S(2)= cox + sin’ x = (cos? x + sin? x) (sin* x + cos! x - cos? x sin’ a) = (sin? x + cos? x)? - 3 sin? x cos? x) & ~3sin?2x . 4 = fee [2.1] . f(x) = 3 cos x + 5 sin (x 7/6) 1 % = seosx+5x —sinx 2 2 Thet, ~ (3) 4 (8) ssoey( y+ (5 = ~Vi9s s(x) sVi9 1 cos 2x + sin 2x = 2k => sin2x-cos2x=2k-1 => sinQx~a) > teh gp gin cot(a+ B)=0 => a+ B= 2+ nn,nel =} sin(a+2B)=sin(90° + B)=cos B (for n=0). We have downloaded from jeemain.guru Trigonometric Ratios and Identities 2.95 Therefore, each ratio is equal to xtytz xtytz => xtytz=0. 21.b, Since 0 0Ssin*" x + cos?" x < sin?x+ cos? x= 1 => OSsin™x+ cos xs 1 22.b, 4-25 x+1=0 Let cand B be the roots, we have ni 8 arp= 28% pat since sin 18°= Y3=!, cog 360 = 5+! 4 4 sin 18°+c0s 36° = “> Bis 5. sin 18° cos 36°= Here the required roots are sin fis cos 36°. 23.b. 2 cot a+ —t = 2 cot a + cosec” sin? = 2 cot a +1 + cot” o@ =|1 + cot a| =-1~cota {since cot @<~1 when 3n/4 < a< m1 + cot a|=- 1~cot a] 24.4. HO=Sexs0+3eas (022) +3 «5cos0+ 3 e028 sinors 2 cose 38 noes a = pe 2) sin(0— a) +3. Thus, the range of f(@) is (~4, 10). 25.b. Since a< P< y< Sand sin a= sin B= Sin y= sin 5= K, therefore B= x— a, y=2n+ a, 5=32- a B 1X 1 sii 8 a a a a in = +3sin =. Zasin= =4 si =-2sin=-cos— => Asin 543sin> qtsing =4sin 2 Tey asin cos +2sin. sin $+ 2c0s$ = 2i4sing = 3VIFK downloaded from jeemain. guru 2.96 ‘Trigonometry 26. ¢. Let O be the centre of the circle Fig. 2.37 Since 240A, = = =60° AgOA, is an equilateral triangle, we get 4g, = I [radius of circle = 1] Als0 dos = Ag = 20D = 2[04] sin 60° = 2(1) (AoA) AoAs) (oda) = V8)) = 3 -27.d. The given relation is satisfied only when sin 6, = sin @,= sin 8 => cos 6, =cos 8 = cos 6 =0 = cos O, + cos 8, +cos A= 0 28.c. sin? @S1 a gy? ee ar 2x => (x-1P+P<0 ’ Itis possible, iffx~1 and y=0. sox tyP-2r+150 , * [asx>0} 29.0. sin a+ f)= 1 = a+ B= & sin(a-B)= 2a ~ B= = a Solving, we get a= 2/3 and B= n/6 x Now tan (@+28) tan (20+) = tan & #5 30.a. sin 27°-sin 63°=—2 cos 45° sin 18° (8) downloaded from jeemain.guru ‘Trigonometric Ratios and Identities 2.97 3l.c. cosec @—cot O=q @ 1 cosec 6+ cot A= — q cone = za +(1/q)] (on addition). 32.a, Squaring both the sides, we get i 1 1+sin20= 25 nom 24 . = sin26=— 55 2 24 Let (= tan 6, we get =912P +2514 12= =9(41+3)G1+4)=0 (=~ 4/3 (as for =~ 3/4 (rejected) as if tan =~ 3/4, then O¢ [n/2, n)and sin 0+ cos 8=~ 1/5) 33.€. Multiplying x above and below by I cos 6+ sin 8, we get 2sin @(1—cosO+sin@) _ 2sin(1-cos@+sin 8) (1+sin@)* -cos*@ (1+ sin 8)? -(1-sin? @) 2 2sin@ 1-cos+sind Putting 1 ~sin? @= (1 + sin 6) (1 ~sin 6), we get == SP wx 34.b, 2nO= m2 8, 2n—1) 0= (n/2)— 020, (2n-2) 0=(72)-26, ... They form complementary angles A and B so that tan A tan B=tan 4 cot A= I for each pair. 35.a, N‘= 2[(sin 1° + sin 89°) + (sin 2° + sin 88°) +--+ (Sin 44° + sin 46°) + sin 45°] ne => [> =2 sin 45°[2(cos 44° + cos 43°+~- + cos}*)] +1} =2 sin 4s? = 36.b. sec a+ cosec a= p, sec acosec a= q downloaded from jeemain.guru 2.98 ‘Trigonometry. xsl in? _ — sin? x'=> sinx = cos? x Now cos!?x +3 cos!°x +3 cos’x +cos®x~.2 = sin*x +3 sin’ x+3 sint x + sin’ x-2 in? x)3 + 3(sin? x} sin x + 3(sin? x) (sin x)? + (sina) -2 = (sin?x + sinx)-2=(1))-2=-1 38a, cos(d~B)= 2 = ScosA cosB+5 sin A sin B=3 < / 0 From 2nd relation, we have sin 4 sin B=2.c0s A cos B (i) => cosAcos B= ; and sin Asin =2 (1+ tan A) (i + tan B) => lnd+tan B=1 39. e fan A tan B => tan(A+B)* Iie, AtB= = ’ n orataaz = ie,a= = 4 20 a Bt 40. & A= += = V2 sin 90° a Ante v2 sin B= Vil Fyne = ¥2sin89° B=9(a) cosa] V2 sin (45°+44°) 41.4. 405 cos 23°~sin 23°) F(e0s30° cos 23°—sin 30° sin 23°) = feos (30° +239) = ; cos 53° in( 182 gin ( 1A 42.6. wn 252) in( 224) teh s, _ £08 8; ~ C08 8, cos 8 +608, 3 a + 08. 80° cos 20° sin 80° sin 2 2sin 80° sin 20° +(cos80° cos 20° +sin 80°sin 20°) cos80° + £98. 20° sin 20°cos80° +cos 20° sin 80° sin80° ~ sin 20° 43.b. + =005100° +c0s 60° +.cos 60° sin 100? — 2sinor sin 7 44.a. tan B=2sin asi rc (a+ ——s A= Rsinimsioy cunalat ye Ey sin (a +) ) => cot B= 2sin asin 7 downloaded from jeemain.guru Trigonometric Ratios and Identities 2.99 sin oF cosy + cosarsiny sin asin => cota, cot B, cot y are in A.P. 45.b. 3 sin A cos B=sin B cos A => 2cotp =cotatcoty = cosdsing=2 4 => sin(A+B)=1 = cmt pata | = 3tan4=tan|*~A on (3 ) = 3=cotA 4 tan 100° + tan 125° tan100°tan 125° = £an 100° + tan 125° tan 100° +tan 125° tan 225° = ——_—___—_—_ ic, | = ——_____—__ 1-tan 100° tan 125° 46.4. tan(100° + 125°) 1—tan 100°tan 125°" ie., tan 100° +tan 125°+ tan 100° tan 125°= 1 tar 20° + tan 40% = tan 20° tan 40° 47., We know that tan (20° + 40°) tan 20° + tan 40° = B= Tian 20° tan 40° => V3— V3 tan 20° tan 40°= tan 20° + tan 40° => tan 20°+ tan 40°+ V3 tan 20° tan 40°= 3 WB ~ V2 cos(a~ 2) ° “f sin{a-4) _ ¥2(1 ~ cos 6) nero En sin? @/2) 8 fa Pisin ee OT 8 4 = 2 sin (0/2) cos 72) 2 (g- z) 49.), sin 6,~sin 8, = a, cos , + cos 8, = b => a +b?=2+2c0s(0, + &) => Osa+osa downloaded from jeemain. guru 2.100 50.¢. 51.d. $2. b. 54.3. 55.b, ~ Trigonometry : 2 2 2 L+sin2x _ (sinx+cosx)* _ stuns) - tan(E +s] = an'(E4) I=sin2x (sinx-cosx)* 1=tan x, 4 4 = cot?{ 2424] = cot” *4,3) 24 4 4 We have 4:2 — 16+ 15<0= dened Therefore, the integral solution of 4x? — 16x+ 15 1 +tanztan e+ tanxtan y+ tan x tan y tan z tan ¢+ 1~tanz tan (—tan x tany + tan x tan ytanztan/=0 = tanxtanytanztant= S(n) = 2 cos nx =f (I) F(a 1)-F(n) = 4 08 x 08 (1+ 1) x-2.¢08 nx = 2[2 608 (12+ I}x cos x ~ cos rx] =2[cos (n+ 2)x + cos nx—cos nx] =2 cos(n+ 2)r=f(n+2). fan _1 _, sin AcosB _1 tanB 3 cosAsinB 3 Put sin 4 cos B= i => cosdAsinB => sinC=1=sin 2 . = C= 72, Hence, the triangle is right angled. 3sin?A+2 sin? B=1 => 3sin?A=cos2B Also 3 sin 2.4 -2sin2B=0 => sin2B= Sin 2A downloaded from jeemain. guru Trigonometric Ratios and Identities 2.101 ‘Now, cos(4 +2B)=cos 4 cos 28-sin A sin2B=cos A 3 sin? A-sin A 3 sin 2A =3 sin? A cos A—3 sin’ A cosA=0 __cot a 1 1 ASM" Te cat a Teta a MMMM OMIA)= 5 x x $7.¢, A—B= = =»tan(4—B)=tan = c. 4 stan (4—B)=tan = tan A—tan B 7 ¥ tan A tan B => tan A-tan B~tand tan B= 1 => tan 4—tan B-tan A tan B+ 1=2 => (1+tan A) (1 ~tan B)=2=9y=2 Hence, (p+ 1)’*!=(2+1)**!=(3=27. 58.a. Applying b- a= c~b for A.P,, we get 2 cos z sin (x—y) = 2 cosx sin (y—2) Dividing by 2 cos x cosy cos z, etc., we get tan.x~ tan y = tan y= tan z. 59.b. (cos a+ cos B)*=(sin a+ sin BP =0 => (cos? a+ cos* B+ 2 cos acos B)—(sin? a+ sin? B+2 sin asin B)=0 = 082+ cos 28=—2(cos,cos sin crsin B) =-2c0s(a+ B) 60.a. We have sin asec x, sec xp + sin @ sec x, Sec x3 +++ + sin Orsecx,_ 1 SCX, = Sings) — a) , sin(sy—ay) | ints, ~ 45.4) COS, COSX, COSX,COSK COS Ky. COS, = (tan x2 - tan x)) + (tan x3 —tan x2) + --- + (tanx, — tan x,_) sin(%—%) _- sin(n—or COSx, COSX, —COS.X,, COS.X, - = tan x, —tanx,= xy + (n= 1) 0} downloaded from jeemain.guru 2.402 ~ Trigonometry 61.a: By the given conditions tan ee tan = =2x tan 2 stan Za 2y 9 18 = 2x=tan 20° + tan 50° __ sin 20° sin 50° cos 20° ” cos 50° = Sif 20°cos 50° + cos20° sin 50° cos 20° cos 50° _ sin 70° £60820" cos 50° Is 1 = = = a cose 40° 2y= tan 20° + tan 70° = sin 20° 70° cos 20° cos 70° sin90° cos 20° cos 70° 0s 20° cos 70° ~ c0s20° sin 20° 2 2 = Fsim30" cos 20° ~ sin age” 260800 40 dy =2Qx=y=2x in( = in(2°—1°) _ sin(Q°=29 in(45° = 44° cap, eg] sin 0, sin2=19) , sinGe—28) |, sas 44°) sin1*| cos0*cosI® cost®cos2°, cos2°cos3° cos44° cos 45° = saan 19+ (tan 2° = tan 1°) + (tan 3°~ tan 2°) + (tan 4°— tan 3°) +---+ (tan 45°— tan 44°)] sin sink? x 63.a, We have sin a+ sin B=— 65 60s a+ cos B=- 2 Squaring Eq. (i), we get sin? a+ sin? B +2 sin orsin B= (2) Squaring Eq. (ii), we get cos? a+ cos* B+2 cos acos B= & zy Gi) Gi) (iv) downloaded from jeemain.guru ‘Adding Eqs. (iii) and (iv), we get 2 +2 cos (a—B)= —— 1 [eon + (65)" = 242c0s(a-f)=—> (6 5 = +005(0- = sinx+y) _atb Sin@—y) a-b 64. sin(x + y)=sin(x= y) sin@x+ y)-+sin(x=y) _ ‘Trigonometric Ratios and Identities 2.103 1 1° ] = (729 + 441 +(y] ( asf ) = (1170) = eae as 2 oe ht } <0 (a+b)+(a-b) (a+b)-(a~b) 2sin.xcos y 2cosxsin y 2a % 30 + sin 50 + sin 70 +sin 96 _ (sin 36 + sin 96) + (sin 56 + sin 76) * cos 30 + cosS@ + cos 78 + cos 98 (cos 30 + cos 98) + (cos 50 + cos 70) in 66 cos 36 + 2 sin 68 cos @ __2 sin 60 (cos 30 + cos 8) _ 65.c. 66.b. 67.c. cos 50°=cos? 25° — sin? 25° = (cos 25° + sin 25°) (cos 25° ~ sin 25°) = p(cos 25° ~ sin 25°) ‘Now (cos 25°—sin 25°)? + (cos 25° + sin 25° = 1 +1 cos 25°~sin 25°= f2—p? We have taken + ve sign as cos 25°> sin 25°, therefore cos 50°= py/2— p® , by Eqs. (i) and (ii). @ Gi) a 68.. A-sin’ B _ 2sin(A+B)sin(A~B) "sin AcosA~sin Bcos B sin2A—sin2B _ 2sin(A+B)sin(A-B) © 2sin(A~B)cos(A+B) = tan(4+B) 0s B B 2 sin?(B/2) 2 sin (B/2) cos (B/2 3= cos 40 + 4 sin 20 2 On subtracting, we get b= (1 ~sin 26) = ab=cos' 20s 1 tan 3 =>tan24=tanB . On adding, we get a= = (1+sin 26) downloaded from jeemain.guru “2104 71. T2a. 73.b. 74.b. cot 70° +4 cos 70° Trigonometry cos? 10° = cos 10° cos 50° + cos? 50° = ; [1 +e0s 20° -(cos 60° +c0s40")+ (1 +c0s 100°)] - 3h #60520? 5 —c0s 40° 10s 80°] i [3+ cos 20° ~ (2 cos 60" cos 0")]=3 2|2 4 tan 20° tan 80° cot $0°=tan 20° tan 80° tan 40° . =tan 20° tan(60°—20°) tan(60° +20°)=tan 60°= V3 tan? @=2 tan? +1 => | +tan? @=2(1 + tan? ¢) => sec? @=2 see? 9 = cos? =2cos? = 1400820 => cos 20=cos? g—1 =-sin? sin? 6+cos 20=0 = 60870" +4sin70°cos 70° sin 70° 08 70° + 2sin 140° sin 70° = 608 70° + 2sin(180°-—40°) sin 70° in 20°-+sin 40° + sin 40° sin 70° 30°cos 10°-+sin 40° sin 70° _ sin 802+ sin 40° sin 70° _ 2sin.60° cos 20' sin 70° 78.b. acosx+bsinx=c o( 1-an?2) 2btan= = +44, 2. x 1+ tan? = I+tan? = 2 a 4B downloaded from jeemain.guru ‘Trigonometric Ratios and Identities 2.105 = (cra) tan? 2—2buan% +e-a=0 cra = tan 2b an 22 and tan 2 ean 22 2002 eta 20 2 cra +yi-x l+Vlex Ifx=cos 6, then Ji=x = V2 sin(/2), Ji¥x = V2 cos (6/2) 76.¢. tan 1 8 r Be eels an aptsng| fing tHe => tany= => 4y= 5+0 => sindy=cos @=x 77.4. We have cos x = tan y => cos?x=tan?y secty—1 cote—1 = 1 +c0s*x= cote tan? x = tan? x cos? x=sin? x => 2sintx-6sin?x+2=0 35 2 = sin’x [': cos y= tan z, see y= cot 2] * cosz=tan.x] downloaded from jeemain.guru 2.106 ‘ Trigonometry es) & => sin?x > sine = 2sin 18° 78.2. sin20=cos30 — => 2 sin Ocos-6=4 cos’ 8-3 cos 8 => + 2sin 0=4(1 —sin? 6)—3 = 4 sin? 6+2 sin @-1=0 _ 5-1 4 => sin 79. b tan. O= A, we get tan? 0/2 = Aton? 2+ 21a 2420 2 = = tan 2 tan % Zz 2 2 80. a tanO=Vn — =900820= 128 _ rational. ‘ 7 SI.b, sin.x+c0s.x= x_4tJi6-47=4 => tmi= x 2 W742) iT+2)2~ 7-2 3 Therefore, the given value = afin? a sin? x = afin? ; + cos? ] ve sin = sin = 2192 |v sin = 2) fas x(cos? x-+sin? x) * 83.b. 4cos?/Z—* 4 fasin® x-+sin? 2x = 400s” 42 S084 = a[t+seo(Z-})+2sinn =2+2sinx—2sinxasxe (= =) =2, downloaded from jeemain.guru ‘Trigonometric Ratios and Identities 2.107 84. bcos’ x sin 2x = cos? x cos x sin.2x _ (Le00s2r)/ 2sin 2x cos x 2 2 = 4 (1~cos2s9(sin3x sin) = fein 3x ett sta! 3x.cos 2x) = A. (2cos2x sin x) alin teins 3 j ne oA priests Uganda 4 2 2 " DL gicorud as 1 at ea = [sn sin 3x- snss| => y= V4, a5= 1/8, n=5 85.b. Given expression is2 sin? $+ 4 cos(8+ 9) sin Osin $+ cos 2(0+ ¢) 86. = (1 —cos 24) +4 cos (8+ ¢) sin Osin 6 +2 cos? (A+ g)—1 =~ cos 29 +4 cos (8+ g) sin Asin p+ 2 cos? (8+ 9) == cos 29 +2 cos (8+ 9) [cos(@+ 9) +2 sin Asin g] 0s 29+ 2 cos (8+ $) [cos Geos 9+ sin Asin 9] cos 29 + 2.cos (8+ 9) cos (8-9) 0s 29 + cos 20+ cos 26= cos 20 £0528 _ cos(A+C) 1 cos(A—C) Applying componendo and dividendo, we get 1-cos2B _ cos(A~C)-cos(A+C) 1+cos2B cos(A—C)+cos(A+C) 2sin? B__2sin AsinC 2cos?B 2cos AcosC tan? B= tan A tanC tan A, tan B, tan Care in GP. uu 1+tan? y/2 downloaded from jeemain.guru 2.108 Trigonometry = Gta? y2= 2tan? 5 => tan Feo = V5 A. & eben att) fet =) “et (<3) 2a 2 ‘ i * Yi-@1a%} __2c0sx Yorn tan? x) ~ Yeeos? x~sin? x) 2cosx (cos2x) 89. h Since aris a root of 25 cos? @+ 5 cos 8-12 =0 2Scos?at+Scosa-12=0 =cosa= ——Y————— =- = 3 and sin a= = 2; mherefore, sin 2a=2 sin acos a=2 C 90.¢. We have tan 9°—tan 27° —tan 63° + tan 81° = (tan 9° +tan 81°) — (tan-27° + tan 63°) 1 1 sin9°cos9® sin27°cos 27° 2 2 sini8° sin 34° _ [eect] sin54°sin 18° 2cos36°sin 18°] _ sin]8°cos36° 91.e. Let A=sin"a, B= sin”! and C=sin“'c, we have A +B+C= 7. ayi-a? +byi-B +evi-c? = Hoin2A-+sin2B +sin2C) = Leasin Asin Bsinc) =2abe 92.d. cos 24 +cos 2B + cos 2C= coe HG cone B)+ cos 2C 4 =2¢0s (F-c}cosa-p) sos2e =—2 sin C cos (4-B)+1-2sin°C 1—2 sin C (cos (A -B) + sin C) 1—2sinC (cos (4B) + sin (3x72 —(4-¥B)} = 1-2sinC [cos (4~B) cos (4 + B)] downloaded from jeemain.guru Trigonometric Ratios and Identities 2.109 =1-4sinA sin BsinC 93. —-=—— A => tanStan 2 => cot 4 cot : 2 2 94.b. sin? A sin? B + sin? C= sin(4 + B) sin(4 ~ B) + sin?C = sin C (sin (4 —B)+sinC) = sin C (sin (A —B) + sin (A + B))=2 sin A cos Bsin C 2 2 2 tan? tan? tan?y 95.¢. sin? a+ sin? B+ sin? y= J+tan? oe 1+tan?B t+tan? y ia + Itx Ity Itz [where x= tan? a y= tan? B, z= tan? 9° ww Gy ta) + Gy yet ee 4 2g2) tay tye Fart, (1+x)(1+ y)(1+z) _lextytetays yetortaye _ Gd dee) (e aptyztixt2oz= 7 Be 2% Also sin A + sin B+sin C= 4e0s4 cos? cos 22 2 sin Atsin B+sinC _ eaten . - sin Asin B=sinC 2" 97, SM2AtSIN2B+sin2C _ dsinAsinBsinC 4. A. BC sin A= 2sin4cos4] inAtsinBsinC 4.4 A oe Boge a a 22 a2 2 downloaded from jeemain.guru 2.110 98. ¢. 99, a 100. a, 101.¢. 102.4, Trigonometry We have cos? A + cos” B- (1 cos? C)=0 => cos’ A +cos’ B-sin?C=0 => cos? d+ cos (B + C) cos (B-C)=0 = 20054 cos B eos C=0 Hence, either 4 or Bor Cis 90°. Ina triangle, tan 4 + tan B+ tan C=tan A tan Btan C @ => 6=2tanC => tanc=3 Also tan A+ tan B=6—3=3 Gi => tan A and tan Bare roots x? - 3x + 2 =0 by Eqs. (i) and => tan A, tan B=2, 1 or 1, 2andtanC=3 We have tan 6° tan 42° tan 66° tan 78° = tan 6° tan (60° ~ 18°) tan (60° + 6°) tan (60° + 18°) tan 6° tan(60° +6°) tan 18° tan(60° — 18°) tan(60°+18°) __ tan 6°tan(60°+6°) tan(3x18°) tan 18° tan 18° tan 6° tan(60° ~ 6) tan(60°+6) _ tan18?_ tan 18° tan 18° Fig, 2.38 s - x Inthe graph of y= sin x, let A =(a, sin @), B= ( sn®) Clearly, slope of OA > slope of OB, so 2% (a—B)=(8—B)~(8- a) = c0s(ar— B) = cos (0 B) cos (8— a) + sin (8— ) sin (@— a”) z z a dykes a=. 2 Baty ay 74 => [2 —cos(a ~B) ab downloaded from jeemain.guru Trigonometric Ratios and Identities 2.111 xy? » x xy = teotta- B)-22 costa Bn Sa a al 2. 322 cosa f)= sin 8 103.b. Let cos 0 {sind + Vein? @-+sin? a} => (w—sin 8 cos 6)? = cos? 6 (sin? 6+ sin? a) => tan? 6-2u tan O+12-sin? = 0 Since tan Bis real, 40? - 41(1? - sin? a) 20. => wWsitsinr-o => |dsvitsin2a 104. a, sin 6, sin @ —cos 6, cos @ => 0s (6,+0,)=1 => 0,40,=2nt,ne! Bann Thus, tan cor % = tan cor(ne-2) =~ tan 2 cor 2° 2 2 2 22 105.a. tan + 2tan 2 ¢4tan = +81an = 3 3 3 wtf saune-t tle )+8tan( 30-2) 3 3 3 3 a x x 2 x = tan -2tan +4 tan —8tan = =—Stan. 2 =-: lan 3 2tan 3 a Stan 3 Stan 3 SV . 3tan 6 —tan? 6 106. ¢. Since tan 30= 1-3tan?@ nae 3tan——tan’ ~ ng 9 1-3tan? = ant 2 = a{i-saZ) = (stn tan? zy . oz am 27 => tan$-33tan* +27 tan? —=3 9 9 9 Putting 0= = , we get tan = 107. ¢. cos x + cos y—cos(x + y) = ; x+y Regs | SB > cos( 2 downloaded from jeemain.guru 2.112 “Trigonometry . ono Noweos( 242) is always real, then discriminant 0. = 108. a, asinx +6 cos (x+ 6) + bcos (x-O=d = asinx+2bcosxcos 0=d => [ds Va" +40? cos? 2g? d > $cos"@ ae la => cos. > ———— leos.6, 21 c , y 4 too. S21 cost 3 tmx 4 ee yyy tamattany tan x siny 2’ cosy 2 tany 3 i-tanxtany 1~3tan? x Alioainy® Qslitx, coup Feosx : . 4co => sin? y+cos?y=4 sin?x+ =1 36 tan?x+4=9 sec? x=9(1 + tan?x) 27 tan? x=5 4% = tanxs = 3V3 uu M10.a, Ji-Feos. 4 Vitcosx +Vi—cosx vi ¥cos.x-Vi—cos.x downloaded from jeemain.guru ‘Trigonometric Ratios and Identities 2.113 6H =ntanty — AM.2GM) —tanty (+ntan? y? in => secXx-y)S1+ n=? (ont? 4n 4n 112. b. cot? x = cot (x—y) cot (x2) 2 cot xcot y+] (2) => coxa | ACO ore cot y-cotx J\ cot z—cotx cot?x.cot y cot z~ cot x cot y— cot? x cot z + cot x =cot?x cot cot z + cotx cot y+ cot x cot z+1 cot? x (cot y + cot z) + cot x (cot + cot z) + 1-cot*x=0 cot x(cot y + cot z) (1 + cot? x) + (1 —cot” x) (1 + cot? x) =0 [cotx (cot y+ cot 2) + (1 —cot?x)] =0 u cot? x. 2eox 2 uoggy (cot y +cotz) = cot 2x downloaded from jeemain. guru 2114 Trigonometry W3.a.4+B+C=n => tanA+tanB+tanC=tan A tan BtanC Now, A.M.2G.M. ne > (tan Atan Btan Cy!" tan A +tan B+tan€ 5 an aan Banc)! => (tan A tan Btan C923 1 213° > (fs K > Kite 33 114, d.a? = (a? cos? @ + 8 sin?@) + (a sin? 6+ b? sin’ 6) =a +0? +2ya? cos? 6+b" sin? @ ya’ sin? @ +b? cos? 8 = a@++2 ysin® Gcos? O(a* +b*)+02b?(sin* @-+c0s* 8) =a +b? +2 Jab? (1-2sin? @cos? 4) +(a4 +b*)sin® Bcos” 8 7 =(@ +6) +2 \fa26? + (a2-67)' sin? 0 cos? 0 =(@+B)+2 ut! 7 a_ yal? Max. earn foo al Z ) Min. w? = (a? + 6?) + 2ab (@-¥) Therefore, the dtference=2 Ya°b? + — 1 ~ 2ab = 4aib? + at + BF 2a°B* ~ 2ab = (a? +0?) -2ab = a? 40? ~ 2ab =(a—BF 115.¢. (sin x + cos x)? + k sin x cos x= L sin? 20 = 1tsinae+ £sin2e 1 ssinar[+t] 0 2 For this to be an identity, 1 + z =05k downloaded from jeemain. guru 116.b, kcosx—k cosx +120 Vx€ (2s, 2) = K(cos*x—cosx)+ 120 Now cos*x—cos x= (= x > 4 Sc0s'x— cos 52 We have 2k + 120and -Eaie0 1 Hence,-— Sks4, a 117.4. min (2+ sin x~cosx)=min [2+visn(«-4)] =2-V2 a b 118. b. a cosec a-b sec a=—— sina cosa ‘Trigonometric Ratios and Identities 2.115 Nowshae Ty gives fa? +6? [ la? +b 119.d. cot*x—2 (1+ cor x) + a4 =cot!x-2cot?x+—-2=0 = (cof x= 1)?=3-a To have at least one solution, 3 - a 20 =>a=350 “ae ENS, V1 Integral values are —1, 0, 1; therefore, the sum is 0, 120, b, sin’x + acosx+ a> 1 +cosx Putting x= 0, we have at@>2 =a@+a-2>0 = (a+2)(a-1)>0 =a<-2ora>I Therefore, the largest negative integral value of ‘a’ = -3. downloaded from jeemain.guru Tatt6 bs: ‘Trigonometry 121.4. dab = ab=60 8 b=ccos@a=csind A ° = A=+ 6? sin 80s 0 = © e a4 Na =>? = 120 cosec2 0 _ o =F pin 120 ; Fig. 2.39 = Cyig™ 230 122. b. From the figure, x08 (8+ 30°)= 8 and x sin @=1—d 0 led Dividing V3 cos 6= Tog" Sdearing Eq. (ii) and O) 1 putting the value of cot 0, we get? = 3 (4d?—4d+4) axe2t 123. d.a=csin @,b=ccos @ _ 40. +sin26) ©" sin? 28 8 45 ~ )weeo (-27)(2-3)=0, ie, 8U"8 = 348"? = 3 oF 3! 3 orsin ga Bah 4'4 a°°3 = 30°, 60°, 120°, 150° sin? 0= 3. b,c, d. Opposite angles of a cyclic quadrilateral are supplementary. 4, bye. = (a)is correct b.sin26= nes £08 26= bung tan 26= aaa => (b)iscorrect. ¢. tan 30= sin 30 ‘ = (c) is correct, 08 cos 36 d. sin @= 1/3 which is rational but cos 30 = cos @ (4 cos* @—3) which is irrational. => (d)is incorrect, . 5. ayb,c,d, -i3)e(3)-=(2) = © Gin 18° cos 36° 16 WG-0 en “8? ve (Jee( downloaded from jeemain.guru 2118” * Trigonometry Lit S) ade - 1a (B)-1-t-200 4.2008? 2cos? = = 2000 4 = =8 (cos 26" cos 40° cos 80°) = # eQ 6. a,b, d. (sin? x + cos? x)’ —3 sin? x cos*x (sin? x + cos* x) > : Gia citins = 1-34? xeos? > = ae = 93 sin? x cos*x = 1-2sin?2x>0 => cos4x>0 xn Age (ee > re ( 7 a) => dre (ans “ $i 2n + =), ne Z, generalizing now verify, 2 [3-(cos A+ cos B+ cos C)] ~1 (08.4 + cos B+ 00s C) 2 => (isincorrect a, b, dare correct and hold good in an equilateral triangle as the maximum values. 8 abe. : a. tan @tan 2ectan 3a = tan 3a— tan 2a—tan @ always holds good. sin 4a + sin 2a _ 2sin3acosa@_ 1 sin 2arsin 4@ sin 2asin 4a sin@ BRAS. = cosec or(using 2/7 = a) downloaded from jeemain.guru ‘Trigonometric Ratios and Identities. 2.119 => (b)iscorrect alert snl) €. €0s G+ cos 3ar+ cos Sa = - asin asin Also cos 20= cos. d 8cos acos2acos 4a 9 a,b,c,d. va Forxe (0.5) tans (tan x) (cot x(n) b.Forxe (0 3) cosecx2 1 => In(cosecx)20 mp qincosecsx < gln(cosee) “exe (0 3) =ecosxe (0,1) => In(cosx)<0 I 1 Also 4 > 4 e375 “yn pysteor = [e <(4 2 3 4. Forxe (0 2) Since sin. x 2sinad —— where f= tan a i-1 1+r a+? -a-1* 2x2 tana. =2tan2a. Pia 1~tan' => (b)is incorrect. Deh De G+ +d-0? 20+) jar ia TF A . (where (= tan @) - =2sec 2 cos 2a 82% => ()iscorrect. 4. tan a+ cot ¢= ———— = 2cosee 2or cos a sin a => Wisincorrect. 12. qd. 8 sec 9 A ten @ AN, Fig. 2.42 Using property of angle bisector, we get secO__x tan@ I=x sec O = x7 ep rang 786 0 (Gee 8 tan 8)= (1+ tan? 2)? i 13. a,d.y= Sy ey Te an? x seal =1+(2-v3) = 8-4/3 =4(2- V3) -{(é-4]] downloaded from jeemain.guru Trigonometric Ratios and Identities 2.121 Aes] = 16sin? & 2 Is 17 8 14, b,d. tan(a+ B)= — and cosec y= — => tan y= — (a+B= > Yo NIE TS at pry te = (b) is true, Also tan(a4p) =! e+tan 6 1=tanatan B a > tanor+tanB tan atan B = tan artan B+ tan Btan y+ tan y tan a= 1 .b, d. Divide by cos crand square both sides and let tan a= ¢ so that sec? a= 1+? [(a+2)t+ Qa-1)P=[2a+ IP (1+)) PU(a+2P-Qat 1P] +2(a+2) a= 1)t+(2a-1P-(Qa+1P}=0 31 =a) P+ 22a +3a—2)t=4x2a=0 3(1-@) P—4 (1-2) 1+ 6at—8a=0 (1-a) (31-4) +2a(31-4)=0 (3t-4) (C1 =a?)t+ 2a] =0 VUUUUYU = retana= 4 or 30 @-1 a, b, 6 logis logy(sinx + a) >0 = 0< log, (sinx+a) sin 1= BF Let log,(cos 1) = y, then 6” = cos ¢ => B= cos’ = 1 sin’ = 1- 4° => 2y=log,(1—b**) \ . syn Zilog! -*) = log, downloaded from jeemain.guru 2.124 Trigonometry 10. a. Statement 2 is true as it is one of the standard results of multiple angles. Putting A= 7/18 in the formula sin 34 = 3 sin A —4sin’ 4, we get 8x° —6x +1 =0, where x=: statement 1 is also true because of statement 2. 11a, Statement? is true, because each trigonometric function has a principle period of mor 2 and hence 2% is one of the periods of every trigonometric function. ‘Thus /(24)=/(2B) = 2A=2nn+2B, forsomene Z = A=nn+B =} Statement | js true because of statement 2, 12,. From Fig, 2.43, sin3 J in statement 2, statement | is false. Given cosx S'cos a -sinx sina =OVxeR Hence, cos a + cos B+ cos y = 0 and sina+sin B+ siny =0 Hence, statement 2 is true. Now (cos @ + cos 8)? = (— cos y)* and (sin @ + sin B)* = (-sin y)? Adding, we get 2+2c0s(@-f)=1 = cos (a—)=-1/2 Similarly, cos (B- 7) Now0 p-asy-a W?2 and cos (y- a) =~ 1/2 an. 3 Statement | is false. . cos? A + cos? B + cos? C= 1-2. cos A cos Bos C 2x2 2 Leos? 4| Seo Alay 21-24 Letx=cotd :y=cot Band z= cot C = Loot Acot B=1 => A+B+C=nn In statement 1 ah 44 LHS. ; [sin 24+ sin 2B + sin 2C] =2sinA sin BsinC 2 |cosee A cosee B cosec C] =42sinAsin BsinC=LHS. Statement 2 is true as it is one of the conditional identities in the triangle. Intriangle ABC,A+B+C=x ~ . RHS. A, BL CA BC and cot — + cot— + cot — = cot — cot= cot — 2 a 2 2 2 2 ‘Therefore, Inf cor 4 +eor2 + cot ©) = incor4+ incot 2 + incor = 2 2 2 2 2 2 Hence, statement 1 is true. Instatement 2, RLS. =In 1 +In 3 +In(2+ J3)=In(1 ¥3 (2+ V3 ))=In (23 +3)=R.HS. But statement 2 does not explain statement 1, downloaded from jeemain. guru 2.122 19, Trigonometry a,b, d. Let x= cos 8, then 4 cos’@—3 cos 0: sastirsat 930-200 2 = 2n , Sm_17R eiepige St tm ae a) 2n_Sm_\In and = BR AE a ¢,d. sinx cos 20° + cos x sin 20°=2 sin x cos 40® = sin 20° cos x = sin x(2 cos 40°—cos 20°) sin 20° sin 20° sin 20° an x= FTF 40°— cos 20° cosd0° + cosd0—cos20° cosd0? + 2sin 30° sin(-10°) = 20° sin20° sin50°-sin10° 2cos30° sin 20° Stanse eo Peeper me : : oe 1d. 2b. 3.b. (x+y) \=xy Itimplies that for any values of xp (xy # 1), we get a value of z and statement 2 is correct. cos? = cos(2n+ 7-22) =cos(7-22) = cos(0.72) ‘Now 1 rad and 0.72 rad angles are for first quadrant where cosr is decreasing, hence cos} cos7. . tan 4 =tan(n+(4~z)) = tan(4~ 2) =tan 0.86 tan7.5 =tan(2x+(7.5—27))=tan (7.5 -2n)=tan 1.22 Now both angles, c.,0.86 rad and 1.22 rad are for first quadrant, hence tan0,86 1 but tan3 sin @ for ¢ (n/4, 2/2), Hence, cos! sin®, = sind, 29 =sind,=0 = c0s%6), c0s%,, .., 6058, = 1 => cos 6, cos 6, . Now cos 8, + 0s 6; + +++ + cos 8, = m4 means two of cos 8), CoS @,,..., 60S 8, must be —I and the others are 1. Now two values from cos @), C08 8,,..., 608 8, can be selected in"C, ways. Hence, the (n=l) number of solutions is °C = T—. Hence, statement 1 is false, but statement 2 is correct. Let y= 2762 x gids = gBensderdsine Now ~¥3? +4? <3cos2x+4sin 2x < V3? +4? or—-$<3 cos 2x+4 sin 2x<5 mp FS g zBeordeedsinde g 95 605 6, = #1 Hence, the correct answer is (a). Obviously in triangle ABC, tan A = tan(x-(B+C)) = -tan(B+C) tan B+tan€ tan BtanC -1 Ifangle A is obtuse, then tand <0 tan B+tanC tan BtanC-1 => tan BtanC <1 (as Band C will be acute) ‘Thus statement | is false and statement 2 is true, ational number. Hence, statement 2 is true. We know that tani'S° = 2—J/3 which isan i 3tan $* — tan? S* Statement 1 is also true as if tan 5° is a rational number, then tan 15° = 25 should be a 1—3tan’ rational number, which is not true, Hence, tan $° is an irrational number. Obviously, statement 2 is the correct reasoning for statement 1, downloaded from jeemain. guru 2.126 Trigonometry Pe For Problems ! -3 La, 2.b,3.c Sol 1. a. sin @=A sin (a+ B)=A (sin crcos B+ sin Beos @) = sina(1—Acos B)=A sin Bcos a 5 ane + (—Acos B) in _ (I-AcosB)tana _ (I-AcosB)sina ate a ‘Acos B ‘cosa cos B : tana+tanB Bee. ti + B)= ———— eA ES alan AsinB sinB —AcosB cos 8 1 Asin Bsin B : (= Atos B)cos B sin Bcos B +sin B tanaz+tan B Also tan(a+ B)= Ma Bm Tan tan sin@ , sina(1—Acos B) =. 08a Acosezcos B sin? @(1—- Acos B) Acos? acos 1 [Asin cos Bi + sine - Asin arcos B]cosa Acos* cos B sin? a+ Asin? acos B sin acosa Acos B —sin? a For Problems 4—6 4.d, 5.0,6.b, Sol. We have tan (« + 2) =3 tan 30 I+tand _, , 3tand ~tan® @ I=tan® 1-3tan? @ sin BcosB __sinB cos B-Acos? B— Asin? B cos B-A @ i) [from Eqs. (i) and (i)) {from Eq. (ii)} downloaded from jeemain. guru ler _(3-r ' 13 222) turnin => 3-67 +8r— Hence, 5, = sum of roots =f +4 +15 +140 5, = sum of product of roots taken two at a time = -2 Sy = sum of product of roots taken three at time = — 8/3 5S, = product of all roots =~ 1/3 tits = Luddy d Dabts =0 i; a he Pa 8 For Problems 7-9 7.a, 8.b,9.d. Sol. sin a+ sin B=3 = 2sin{ 222 Joo 2=2) =3 cos a+ cos B=4 @+B) (a-B)_," ws son{ taf Bn Dividing Eq, (ii) by Eq. (iv), wehave tan( 222) -3 2) 4 2uan( 228 2x3 2 4 => sin(a+ B= For Problems 10-12 10a, 11.6, 12. b. Sol. Let @= a (so that 70= nz) 9+ 40+30=ne = tan46=tan(nx—36)=-tan 38 4 tan 6 ~4tan? 3 tan tan? @ 1-6 tan? @ +tan‘ 6 1-3tan? @ Trigonometric Ratios and Identities 2.127 downloaded from jeemain. guru - 2428. »«. Trigonometry frre = x6] = 4-42) (1-37)=-G-24) (1-6 +24) => 2-2124+352-7=0 . nO} This is a cubic equation in z%, ie., in tan? 6. sum of the ‘The roots of this equation are therefore tan? 7/7, tan? 2a/7 and tan? 32/7 from Eq. ( 21 = tan? 2 tran? 2% stan? 3% 221 . Gi) 7 7 7 Putting 1/p in place of z in Eq, (i), we get—7)° + 35y4—21y? + 1=0 or 7y8—35)4=21)7?— This is a cubic equation in , in cot? 8. ‘The roots of this Eq. are therefore cot 2/7, cot 2n/7 andcot? 32/7. Sum of the roots of Eq. (iii) = 35/7= 5 2 2n 230 = scot 7 = cot = 7 teat aac (iv) By aie Eqs. (ii) and (iv), we get tan? tan? 2% 41an? 2 ]{ cot? 2 +-c0t? 2% 7 7 7 Gi +cot t 238) 291 x5=105 7 For Problems 13-15 13.b, 14.b, 15.4. Sol. Angles BEC, ABD, ABE and BAC are in A.P. Let ZBEC= 0-38, ZABD= af, ZABE= + Band ZBAC= a+ 5B From A ABD, a—B+ a+ 3B = — downloaded from jeemain.guru Trigonometric Ratios and Identities 2.128 = 2p+28- 7 > a+ Bao Now, a-3B =(a+3f)+ (a+ B) (using exterior angle theorem} = a=-78 ei, B= Fig. 2.45 x x x 28=Aat P= —, ZA=—,2C=— (a+ B) 2 6 3 = ABCisa30°-90°-60" triangle. 2 13. Area of the circle circumscribing A ABC = (3) = ;. By 14, ABOCi: ilateral 42 : ; ee isequilteral r= == 2\2 niin 18. BD=OBsin — = — sin — = — oBsin © => sin = p8’=280- 2 1 a >gborje3s3dop cos @-sin = 2, where 0< <= : 0 ‘Squaring both sides of Eq. (i), we get 1=sinzo= 25 = sin2e- 74 = cosza= 25 25 Also, (cos 6+ sin 6)? = (cos 6 sin 6)? +4 cos @sin @= $ wenden, oS M2 2525 downloaded from jeemain.guru 2.190 5 | sy THgonometry ste = cos O+ sin O= = < @ = (cos 0+sin A/2= z Also solving Eqs. (i) and (ii), we get cos 8= 4/5, 2. aaqib sc>pidor a A=sin? 0+ cos’ @ su28 (econo) 2 £0820+++40820++cos? 20 4.2 4 pi) 31 il eesap 2 Now, -1 $cos40<1 cos 48 1 u Is u (Hs) 3,14 ——— |s=+24+= 2 4°88 u BIW BIW o SAS1 . 2 bh A=3 cos? 6+ sin’ @= st os28 ,Lreos2#) 2 343c0s2t 2c0s 20 +cos” 20 2 4 = 74400820 +008? 26 _ (c0s26-+2)* +3 4 4. Now, I Scos20+2<3 2 = 1 ©2042) +3 « A=sin® @-cos' @ _ 1+00s28_(1+c0s26)* Z 2 ee Gal 26 42 downloaded from jeemain.guru o Trigonometric Ratios and Identities 2.131 $-00526—eos? 26 = -(Jovs'20+os20-1) 4 4 a ~ 3-(Zoos20+1) 4\2 lel 1 Now, ~— S >cos 20S — low, ~> S508 20S > 2 ie = }s(feos29n) <2 = ~2s-(Seosros1) <1 = -1s$-(Leosaost} <1 4.4 = tan? 6+2 cot? = (tan 6- V2 cot 6? +22 22V2 . AIH pegs cos &+cos B= 1/2 > 2eos(% P)co( 2-8) 4 0 sin a+ sin B= 1/3 = asio(# ; Py o( 228) 1 , w Dividing Eq. (ii) by Eq. (i), we get offs) 28) Squaring and adding the given results, we have + wal 8 2+2cos(a ~ B) a6 59 2 Now 2c0s( %=2) 1=cos(a - f) = cos(a- A) > 2eos'(* downloaded from jeemain.guru 2.132 . Trigonometry a inf 4. a>s;bo rc 3 pjdoq” in (720° + 90°)=sin 90° = 1 sin? 2°—sin7 1° sin3°sinI@ 1 2sin 3 sin 1° 2sin 3°sin © 2 c. sin (870°) + cosec (~-660°) + tan (855°) +2 cot (840°) + cos (480°) + sec (900°) =—sin (810° + 60°) —coséc (720° ~ 60°) —tan (810° + 45%) + 2 cot 120° + cos 120+ see 180° d. cos (0-4) = cos Gcos $+sin Asin p= ox Boao temp a5 5 5. a>s;b>rjc>q;d—p a {cos (24 + 0) + cos (2B + 6)} =2cos(4 ~ B) cos (A + B+ 8) ‘Maximum value is 2cos (A —_B) when cos (4 + B+ 6)=1 b. {cos2.A +cos2.B} 2cos (A+ B) cos (A ~B) Maximim value is 2cos (4 —B) when cos (A+ B)=1 ¢. For = sec.x,x € (0, 7/2), tangent drawn to it at any point lies completely below the graph of y= secx, thus series = sec(A+B) = sec2d +sec2B2sec(A +B) Hence, the minimum vatue is 2 sec (4+ B). d, yfftand + cot8-2c0s2(A+B)} = y(viand - Veo 8) +2-2c0s2(A+B) = y(Viand ~Jeor6)" +4sin?¢A +8) Minimum value occurs when Jian@ = Jcoi6 and minimum value is y4sin2(A +B) =2sin(4+B) 6 asssbonespidoq a. cos 20° + cos 80°~ -/3 cos 50° = 2cos 30° cos50°~ V3 cos 50° 5 coss0°— x/3 cos 50°=0 b.c0s0°-+e0s 2 +c0s 2 +005 2 +005 4% + cos 3% +005 % 7 7 7 7 7 7 =1+ (cos%+00&) + (cof +eos52) £ (:os3E +0042) 7 7 7 7 7 7 downloaded from jeemain.guru ‘Trigonometric Ratios and Identities 2.133 . ( tn ( *)) 2n ( a 3n ( a s =1+| cos—+cos| r-— || + | cos—-+cos| m-— || + | cos—+cos| x-— 7 a 7 7 az 7 =1+0+0+0 €..€05 20° + cos 40° + cos 60°—4 cos 10° cos 20° cos 30° = cos 20° + cos 40° + cos 60°—4 cos 10° cos 20° cos 30° =2cos 30° cos 10° +2cos?30°~ 1 ~4 cos 10% cos 20° cos 30° 1c0s 30° (cos 10° + cos 30°) ~ | ~4 cos 10° cos 20° cos 30° cos 30° (2c0s10°cos20°) — 1 4 cos 10° cos 20° cos 30°=—1 d. cos 20° cos 100° + cos 100° cos! 40° cos 140° cos 200° 1 7 (Cos 120°+ cos 80° + cos 240° + cos 40° cos 340°—-cos 60°) ; +c0s 40° —cos 340° ; +05 80° + c0s 80° + cos 40° —cos 20°) 3 + 2cos 60° cos 20° — cos 20°) = 7. a2Qb>q;c> pd ps a Since angles, A, B and C are acute angles “AF B> m2 x arg-8 sin A~cos B>0 = cos B-sind<0 o ini, BR Again, B> —A sin B> cos sin B-cos 4>0 @ From Eq. (i) and (ii), we get that x-coordinates is —ve and y-coordinate is +ve. Therefore, line is in 2" quadrant only b 28" 8> | => sin 02> 0=9 Be 1 or 2" quadrant 38% < | => cos 0<0 => Be 2” or 3 quadrant: Hence, @¢ 2" quadrant ©. |cosx+sin.x|=|{sinx]+|cos.x| => cos x and sin x must have same sign or at least one is zero. => xe 2" or 4" quadrant aur % ae = = which is true only if|cos |= cos A 8. a> psbp;e—>qi;ds a x=sin 8 y= cos 6 P= (3sin @-4sin°? + (3cos @—4cos*6)? = sin?30+ cos?30= I oe sha =(1+sin2g? On subtracting, we get b= (1 ~ sin 26)? => ab = cos*2 <1 c. 3cos =x? 8x +19 => 3cos O=(x-4P +3 ad LHS= bh Onadding, we get downloaded from jeemain.guru 2.134 : Trigonometry Now LHS. = 300s 83 or L.H.S. has the greatest value 3. But R.H.S. (x—4)?+3 23 or HS. has the least value 3. Hence, L.H.S. = R.H.S. when 3cos 0=(x—4)?+3=3 => 00s @= I andx-4=0 ‘ = 0=2ngandx=4, where ne Z d A=tan@ x=2sin 20andy=2 cos 20 E=x2—xy+y? = 4-4 sin 28cos 20= 4-2 sin 40 Ec [2,6] =7atb=8 9. argsb>pjers;d—or a 9+16 +24 sin (4 + B)=37 (on squaring and adding) «24 sin (A+B)=12 sin(A+ B)=3 sinc = vie C=30° or 150° = C=30°(+s forC = 150°) h (sin A+ sin B)’-sin? C=3 sin A sin B = sin? A —sin? C+ sin? B= sin A sin B = sin (A+ C)sin(A-C)+sin? B=sinA sinB = sin Bjsin (A —C)+ sin (4+ C))4 sin 4 sin B =)2 sin A cos C* sin A(as sin B#0) = cosC= 12 3C=60" . ©. 2sinxcosx[4 cos'x —4 sin*x] = 1 = (sin 2x) [2(cos?x + sin?x)}[2 cos*x —2 sin?x} = | = (sin 2x)2x2.cos 2x =2sin4x=1 sind = 5 9dy=30?9x07.5° 4 . Obviously, AEOD is a cycli¢ quadrilateral, we have ZCOD= 120° + 45°= 165° downloaded from jeemain. guru ea 1—sin 20 + cos 20 1.4) SO = Jes 20 (cos 8 — sin 0) + (cos? @—'sin? @) 2(cos 6 ~ sin 8)(cos @ + sin 8) cos 6 A “cos @+sin@ 1 “T+ tan 11-104) 1 L (tant) C+ an 34) 1 fix (14+ tant 1°) Ss (1+ tan(45° = 11°) =_!_y, iL (1+ tant?) ( 1 sit) 1 2 1+ 1+ tan iI? 1 (i+ tant) _ (1+ tan 119) 2 2. (5) fx) =2( 7 cos x + 24 sin x) (7 sinx ~24 cos x) reos6=7; rsin@=24 = 625; tan O= 3 Kx) =2r 08 (x= 8) xr sin (x8) = P(sin2 (x- ) Kena =23? = (fay)! = 5 3. (1) A+ Base 48 = 45° mij bee) Trigonometric Ratios and Identities 2.135 downloaded from jeemain.guru 2.136 ‘Trigonometry 1-tand 4.(5) (1 +tan [1 sonesr—o)-ci+un9 (14 Fas] 2 “coo ira)“ Hence, L.H.S. is equal to 2(1 tan SL + tan 40°V(1 + tan 10") tan 35°X1 tan 152)(1 + tan 30° + tan 20° 1 + tan 25°) =2x2'=2) =2sin(40°)cos(40°) | sin(20°) cos(80°) cos(20°)| _ 3 5@ V3 cot(20") + tan(@0) cot(20°) jtan(20°) — tan(80°) 1 + tan 20° tan 80° = V3 tan(60)=3 6.(1) Letx+5=14cos and y~12= 14 sin @ vixt+y? = (14 cos 6-5)? +(14 sin 8+ 12)? = 196 +25 + 144 +28(12 sin 8-5 cos 8) = 365 +28(12 sin 8-5 cos 6) tty? le [365 28x13 = J365- 364 =1 7.8) cot x +coty= 49 1 +t =49 ri ‘ax tany lany+tan _ 4g ‘an xan y = tanx tan, = tmatimy 2.6 ae “9 49° 7 = tan (x+y) aD a 294 which is divisible by 2, 3 and 7 but not by 5. pO wo ible by 2, eM : : 8. (7) From the given equations, we have (2.cos a+9 cos d)?= (6 cos b+ 7 cos c?? And (2 sin a—9 sin d)? = (6 sin 6-7 sin c)? Adding; we have 36 cos (a+ d) = 84 cos(b +c) coa+d) 7 = ‘cos(b+c) ~ 3 downloaded from jeemain.guru Trigonometric Ratios and Identities 2.137 9. (8) Since cos A + cos B=0 =A+B=n, 2 B=n-A = sin A+ sin(n—A)= | =sind=> = A=30° and B= 150° or A= 150° and B= 30° => 12 cos 60° +4 cos 300°= 8 2unB —_-2una 10.4) ST tan? B 3 14 tan? Stan B 3tan a = T+ tan? 6 1+tan?a@ 0 Substituting tan B= 3 tan @, we have Sx3unq | _3tana_ 149 tan?@ I+ tan? a 54S tan’a=1 +9 tanta =4tanta=4 = tana=l,ie,tanB=3 van a+ tan B=4 11, (4) Let @= pa 8e= qo maro ln 8 FeO ean? 2. y= (lan 8 cot 8) + (tan 58- cot 56) [as tan 130 tan(80+56)=—cot S@and tan 98 tan(80+ 6)=—cot 6) = (tan 6~ cot 6) + (Got 30=tan 36) = sin? @~ cos? @ , cos? 30-sin? 30 sin @ cos @ sin 38 cos 38 = 9). £0860 _ cos 20 sys sin 69 sin 20 2[ sin 20 cos 66 — cos 20 sin 68 . sin 60 sin 26 sin 40 x =-2/—3 fea | | 622-20 [aoa al ( 2 ) Hence, absolute value = 4. 12. (2) cos 290° = sin 20°; sin 250° = — sin 70° =—cos 20° 1 1 = -— + sin 20° 3 cos 20° downloaded from jeemain.guru 2.138 Trigonometry Vacos 20° = sin 20° aBsin 20°cos 20° — Asin 60° cos 20° ~ sin 20° cos60°] V3sin 20°cos 20° Hence, the greatest intéger less than or equal to is 2 13. (4) sin®x + cos®x = (sin’x + cos?x) (sin’x + cos*x — sin*x cos*x) sa diet = 1-3 sink costr = 1 - Mein 28 : 4 4 SP" 4a 3G6in 2 4 You Fax) 4 tart cosx _ sinx 14. (3) cows" 459+ (ins cons | a7 -4] + (sin x ~ cos x)” Baxi 34-c1 = 5 (0-sin 2x) = 3 (-)) 3 Hence, the maximum value is= 5(1~(~1)) =3 15. (6) Nr. = (sin? ¢ + cos? ¢)? —2 sin?¢ cos*¢— 1 =—2 sin®t cose Dr. = (sin? + cos? # —3 sin? cos*¢~ 1 =—3 sin’t cos’ 1 1 1 1 1 1 sin 10° sin 50° sin 70° ~ G53 80° * cos 40° cos 20° 16. (6) __ £08 40° cos 20° + cos 80° cos 20° — cos 40°.cos 80° cos 20° cos 40° cos 80° = 8[c0s20°(cos40° + cos80°)—cos40° cos80°] = 8[2cas20° cos60° cos20° —-cos40° cos80°} =4[2cos*20° — 2cos40° cos80"} =4[1 + cos40° -(cos120° + cos40°)] =4y3 2 downloaded from jeemain.guru Trigonometric Ratios and Identities 2.139 17. (7) f(x) = 9 sin2x— 16 cos?x ~ 10(3 sin x~4 cos x) — 10(3 sin x + 4 cos x)+ 100 = 25 sin?x—60 sinx +84 = (5 sinx- 6)? +48 The minimum value of f(x) occurs when sin x= 1. Therefore, the minimum value of /f(x) is7. 18. (0) In AABC, tan A + tan B+ tan C= tan A tan Btan C sextet] + 1—x=x(1 +x)(1-x) x s=-2ax5-218 =2tx= stand =x<0=> A is obtuse = tanBex+1=1-2'5<0 Hence, A and B are obtuse, which is not possible in a triangle. Hence, no such triangle can exist. sin 2x 19. (4) Given logig () @ I loBi0| 79 Also logyo(sin x + ¢0s x) = —> : n = logio(sin x + cos x)? = logi9 (3) = 1 +sin2e= 75 1 n site 75 Som =3 "0 aie 3 downloaded from jeemain.guru 2.140 Trigonometry 20, (4) 28nd? c0s3° + 2sind? cost? _ sin 4*c08 3°-c051°) ° cos! ®cos 2°sin 4° os 1°cos 2°sin 4° = 4008 2°cosl? _ os 1°cos2° A. BoC I 24. (1) 2sin sin Fsin> = 7 A-B => | cos 2 = sin £(f-an$ za -@@ 1G, i ChE ap = sin? 5 Sain 54 2 w= (t-sngJ =o = sing 2b 2 4 c 17 C=1-2sin?= =1 = = cos sins 273 22, (2) In the triangte, tan 4+ tan B+ tan C=tan A tan B tan = LOk+DGk+) 3 2 2 D +3k 8K? +6K 4+) _ 346k 8 2 = 8+ 6k +1 = 12+ 24k = 8 — 18k-11=0 => 8 ~ 22k + 4k-11=0 => (2k+ 1N(dk—11)=0 For k=—1/2, tan B= 0 (not possible) # 4 23. (4) 4 sin?x cos 3x +4 cos'x sin 3x = ; 3 =9G sin.x~sin 3x) c08 3x + (3 e08.x + 05 3x) sin 3x = 5 Rie => 3fsin x cos 3x + cos x sin 3x] = 1 =sinax=— 2 downloaded from jeemain. guru Trigonometric Ratios and identities 2.141 een Subjective 1. Wehavetan(a+ py= nation. T=tanartan B m 1 = 41" 2ma1 m 1 I- x m+) 2m41 am? + Im? +2m+ = at B=nn+n4, where ne Z. 2.9, To draw the graph of y= Fyn teoss) fois 4 tox= a y yesing y= sin (x + 4) x =n nis 0 mid 2 3nd = x sin x+e0sx) = sin[ x + hh We have cos (a+ B)= : and sin (@—~ B) = tan(a+py= 3 and tan (a B)= a tan(ar= B)+tan(a+B) _ tan 2a= tan [(a+ 8) + (a~f)) = i= tania = Bytancers B) 3, We have, 13 3 5cos 0+3 cos (o+5)+3~5 cos 0+3.c0s Bcos Fo Bsinasin 5 +3 - Boosa—23sino+3 downloaded from jeemain.guru 2.442 Trigonometry 7 7 13y (3v3) 213 w3 2) 33 Now, -, (8) J $0080 === 7 sinds > as Boos 2B nos, 13 33 = 4s Fc0sd-sin +3510 4, Given a+ B= mand to prove that sin? c+ sin? B- sin? y= 2 sin asin Bcos 7 L.H.S. = sin? r+ sin? B~sin? y = sin? a+ sin(B+ 7) sin (B-/) . = sin? a+ sin(B+ 7) sin (x-@) (+ a+ B-7=2) = sin? art sin(B+ Ysin in ce[sin @+ sin (B+ y)] =sin ofsin[x—(B- 7)] + sin(B+ 7) = sin afsin(B— 7) + sin (B+ 7) =sin [2 sin Boos 4 =2sin asin Boos 7 =RHS. 5. We have, cos 0+ sin 0= Va $oor0--Josna| v2: cos 6+ sin O< J2 <2 (24) _ m12=1,57 cos 0+ sin O cos @ sin(cos 6) cos(sin )> sin (cos 8) Gi) 6, L.H.S.=sin 12° sin 48° sin 54° = 12 sin 12°c05 429] sin 54° = Je s—Lsnse] 2 2 - [asin 54°—sin54°] Af ins _(i+vs rir 4 aA of 1+5+2V5)_(1-v5 4 16 4 wdedtoens- 2 “NB = 3x4 = =RHS. zh 8 downloaded from jeemain. guru ‘Trigonometric Ratios and Identities 2.143 7, We know that 1 an fone 08.4 ¢0s 2A cos 44 --- cos 2"4 = ———-sin (2"* A) 2 sin A an 2n 2 (2 3 (2m 1 is — 2| = | cos 2° | — } cos 2° | — 6 cos 2 cos 2{ os (eos 2 sin (24) = 16+ (where A= 27/15) 2 sin A : a 1g Sin (324/15) 16 sin 20/15 sin (322/15) _ sin(32e/15) sin (20 +2n/15) sin (32n/15) 8, Weknow that 2tan a tan2a= + =tan? @ 2 . done. tar = 2 cot 20 = cota-tana=2cot2a . o n Now we have to prove tan cr+ 2 tan 20+ 4 tan 4 a+ 8 cot 8ar= cot LHS. tan +2 tan 2 a+4 tan 4+ 4 (2 cot 8a) tan a+ 2tan2 a+ 4 tan4 c+ 4 (cot4 or—tan 4a) (using Eq. (@] =tan @+2 tan 2@+4 tan 4a+4 cot4 a4 tan4 & =tan &+2tan2a+2 (2 cot 4a) =tan a+ 2 tan 20+ 2 (cot 2 a-~tan 20) [using Eq. (i) =tan @+2 cot 2 @ =tan a+ (cot @—tan a) [using Eq. ()] =cota=R.HS. 9. Given that in’ ABC, A, Band Care in A.P. 2 AtB=2B Alsod+B+C=180° = B+2B=180° = B=60° Also given that sin (24 + B) = sin (C—A) = -sin(B+2C) : : 1 = sin QA + 60°)=sin (C-A)=-sin (60 +20)= 5 0 From Eq. (i), we have . sin(2A + 60°) ; = 2A+60°=150° tan x (1-3 tan? x) ‘ 3 tan x tan? x 3tan? x downloaded from jeemain.guru 2.144 12. Trigonometry ‘ 3y—(tan? xy = 1-3 tan? x (y—3) tan? x= 3y vy (LHS. isa prefect square) = zor y23 ‘Thus, y never lies between 1/3 and 3, fe = Fon-kc0s 7 a (amie 4 a zyenie tn econ o x a n We know that cos @= cos (27-8). Replacing each angle @by 2n~ @ in Eq, (i), we get S*(n~ Neos (n= 1) 22+ (n= 2)cos(n 2) 2 +++ 1eos 2% fusing Eq. ()} «i 7 a 2 ‘Adding terms having the same angle and taking n common, we have 25 nf cos reas scons. -seonen-)] n ” n n sinn—= Za (n=1)2% | —— cos —#_____ = sin= ” =nlcosa==n (+: sin(e~@)=sin 8) Ss’ Given that 2 2sin = LT2E*SE rg (m2, 2/2) 3a? -2x-1 This can be written as (Gsin¢~5)x?+ 2(1 -2 sin) x—(1 +2 sin) =0 . For the given equation to hold, x should be a real number, therefore the above equation should have real roots, i¢., D>0 = 41 -2sin#? +46 sin1~5)(1 +2 sin) 20 = ‘16sin?/-8sin¢-420 => (4sin?s—2sin¢—1)20 downloaded from jeemain. guru = {sin = snr s-( 1) = sin Bt = Sin/Ssin(#/10)orsin/2sin(3n/10) => 1<—n/l0ore23x/10 (Note that sin x is an increasing function from ~2/2 to n/2. Therefore, the range of is [~2/2,—n/10] U [37/10, 7/2) Trigonometric Ratios and Identities 2.145 Bo) sins ot 20 4 4 1 1 a. ———_,—_= A cost 1 +3 sin 20 21 +e0s 20)-+14+ 3 in 20 pes 208 20+ sin 20+3 7 7 Now =, 2 +(3) $2005 20 + 3 sin 265 2 (3) or -2<2.c0s 26 +3 sin20 <> 2 2 2 = 452005204 3sin 2043 57 ; . =2s—_1____22 i 20s 26+ sin 20+3 Hence, the maximum value is 2. Objective Fill in the blanks 1, According to the given question, we have expressed L.H.S. in the form Co + C,cosx + C208 2x + + C, 008 nx. Now, sn resi a:oatnSfynethil sin’ y sin 34= 2SATSINIE 5g, Bsinsesin3x—sin? 3x __3(c082—cos 43) ~(1=cos 6x) 4 4 8 Hence, n=6. 2. We know that A.M. >G.M. {implies that the minimum value of A.M. is obtained when A.M. = G.M. Therefore, the quantities whose A.M. is being taken are equal. downloaded from jeemain. guru 2.146 ‘Trigonometry Fig. 2.49 ‘That is, cos (a+) -0a(+3)pon(r+3) => sina=sin B=sin 7 Also, or+ B+ 7= 360° => a= B= y= 120°= 2013 Therefore, the minimum value of A.M. = ————+4—+==$4 A= a2 _ 2 -3sin— VB 3 3 sin sin 5% sin 2 sin 2 sin sin 1% sin 2 Ta a Ta a ee St) | 3n).. = sin% sin sin = sin sin{ w-2* ) sin{ x= |sin{ x - 4a a2 14 14) 1 = sin? Zin? 2 sin? 4 4 144 oae}T downloaded from jeemain.guru Trigonometric Ratios and Ident -( ! ay 8sinn/7 7 . (sueeneny . ( sanity -() _ 8sinx/7 Bsinn/7) 8. 4 k= sin 2 sin sin 2 18° "18 ne in(x 2/9) = nx] ine ingro=l, n>” ganz eg Bsin— 9 5. A+B=a/3) => tan(dt+B)= V3 tanAttanB tan Atan B tand+—— = = [where y=tan 4 tan B} Ny : => tan?A+ V3(y-ItanA+y=0 For real value of tan A, 3-1 -4y20 => 37-1yt320 = o-a(v-t}e0 > ys oryes But A, B>Oand A+ B= 2/3 =A, B land tanB<3 YS x.Le,, the maximum value ofyis 1/3. 1 & Wehave 2. — 1. Sit cosx y) COs(x+y) cost x—sin? y = cos?x=2 sin? y = cos? x cosy downloaded from jeemain.guru 2.148 ‘Trigonometry = cos’ x(1—cosy) =sin?y ~ 242 sin? Leasin? 2 gos? 2 = cos’ x2 sin —=4sin° —cos* = 2 a oi => =2e0%2 2 => cos? x sec? 2 2 > cos x sec True or false asin? 1~cos B 1. tana = SS gry sinB Boge 2 2 B : 2tan 2 Hence, tan 24= 284-2 stan Ytan™A joan? 2 ~ 2 Therefore, the statement is true. Multiple choice questions with one correct answer 1.d, From the given relations, m +n=2 tan @,m—n=2sin 0 => mP-=4 tan Osin@ Also 4 Vinn = 4Vtan? @—sin? @ = 4sin 8 tan@ From Eqs. (i) and (ii), we get m*—n? = 4/mn. 2b. tan 9= = => G¢ Iquadrant or IV quadrant “= sin @= 44/5 If Be Il quadrant, sin 0= 4/5 If Be 1V quadrant, sin @=— 4/5 3a at Bt y=2n= S82 tan($+8) = n(x tan a/2+ tan B/2 1=tan@/2tan B/2 => tan @/2 + tan B/2 + tan y/2 = tan @/2 tan B/2 tan y/2 4.b. We have sin? 0+ cos" @= sin? 6+ cos? cos? @< sin? 6+ cos” 6 Thus, A* sin? 0+cos* <1 ’ n =tan 7/2 @ @ [+ cos? 0s 1] downloaded from jeemain, guru Trigonometric Ratios and Identities 2.149 Again, A= sin? 0+ cos! @= 1 —cos* 0+ cos* = 1 + (cos! - cos? @) =1+(as?o- Hence, 3/4$4 $1, 5.c. We have cos a = cos («-§) =~cos = 8 8 8 8 LHS. = ( + cos a\) +cos = — cos 3) ( — cos 8 8 a) - (: — cos? 2) (i = cos? =) 3 Sn = sin? 2 sin? 8 8 ~s(em sem") “4[(-=3)6-=3] {aes 6.¢. The given expression is V3 cosec20° — sec20° = sin20° cos20° _ ¥3cos 20° —sin 20° sin 20°cos 20° B %3 os 20°—1 sin 20° Seka sanil 2sin20°cos 20° 4 sin 60° cos 20°—cos 60° sin 20° sin 2x 20° = 4sin(60°~ 20°) _ 4si in 40° Te. 3 (sinx—cosx)! + 6(sin.x + cos.x)? + 4(sin® x + cos®x) =3 (1 —sin 2x)? +6(1 + sin 2x) + 4{(sin? x + cos? x) —3 si x cost (sin? x+cos?x)] =3(1-2sin 2x si? 23) +646 sind) 4[1~3sin?2s] = 13+3 sin?2x—3 sin? 2x= 13 downloaded’ from jeemain. guru 2.150 ‘Trigonometry any (x+y)? 8b. Given, sec? 0= Now, sec? @ > 1 => 21 (x+ of <> Gey? Sry = (etyP-ays0 => @&-yls0 But for real values ofxandy,(x-y?20 => (x-y)*=0 oe Also ayo x80. p40 9. ¢. f(@) =sin 8(sin 8+ sin 38) = (sin 8+3 sin 6-4 sin? 8) sin sin? @(| —sin® 6) = 4 sin? @.cos? = (2 sin Ocos 8) which is true for all @. 10.a, We are given that (cot'a%) (cot a) ++» (cot a,)=1 = (Cos a) (cos a) +++ (Cos a,) = (Sin a) (sin a) (Sin a) 0 Let y= (cos 0%) (cos a) +++ (cos @,) (to be maximum) Squaring both sides, we get = (cos? a) (cos* a) -- (cos) = CoS Gy SiN GCOS 0} Sin G, +++ COS A, SiN Oy [using Eq. (i)) (4 sin 0-4 sin? 8) sin @ in 20° 20 = lsin29, sin 2a, ---sin2a,,] AS0S@, Op, ~, 0,5 0/2 0524,20,-,20)S% => 0Ssin2c, sin20,-,sin2a, <1 ys #1 =ys ‘Therefore, the maximum value of y is 1/2", pe Ie. a+ BoE = an Fp => wnaxcotB => tanctanB=1 0 Again, B+ y= a=> y= a-B tanor—tan B 1+tanatanB = tana=tanB+2tan7 12.b, Given that sin = 1/2 and éos $= 1/3, and @ and ¢ are acute angles. = nib ando< teh we ee ana — tan B = tany= {using Eq. ()} 3 asters or 5 <046 <2 downloaded from jeemain. guru Trigonometric Ratios and Identities 2.151 a 1 2sin?{ =x 4 4 r ———— ee = tan| ——x a “ x 4 x} sin{ 2—xJeos| £—» 13.b. sec 2x—tan 2x = 14.4, sin 2 +005 sin? 2+ cos? 2 +2sincos= = 2 se sm 8 2 2n non 4 z => I+sin— n For n= 2, the given equation is not satisfied n Considering that n> 1 and n#2,0

You might also like